Журнал вычислительной математики и математической физики, 2021, T. 61, № 9, стр. 1431-1446

Условная оптимизация функционального вычислительного ядерного алгоритма приближения вероятностной плотности по заданной выборке

Т. Е. Булгакова 1*, А. В. Войтишек 12**

1 Новосибирский государственный университет
630090 Новосибирск, ул. Пирогова, 2, Россия

2 Институт вычислительной математики и математической геофизики СО РАН
630090 Новосибирск, пр-т Акад. Лаврентьева, 6, Россия

* E-mail: tatyana.bulgakova@gmail.com
** E-mail: vav@osmf.sscc.ru

Поступила в редакцию 18.08.2020
После доработки 29.12.2020
Принята к публикации 07.04.2021

Полный текст (PDF)

Аннотация

Рассмотрена задача получения численного функционального приближения вероятностной плотности по заданным или моделируемым выборочным значениям с заданным уровнем погрешности и с наименьшими затратами. Для решения этой задачи предложен вычислительный алгоритм, представляющий собой функциональную версию ядерной оценки вероятностной плотности. Этот алгоритм аналогичен функциональному вычислительному ядерному статистическому алгоритму приближения решения интегрального уравнения Фредгольма II рода, для которого в свое время была построена теория условной оптимизации. В данной работе эта теория строится для построенного функционального вычислительного ядерного алгоритма приближения вероятностной плотности. Библ. 27.

Ключевые слова: численное функциональное приближение вероятностной плотности, численная аппроксимация функций, функциональный вычислительный ядерный алгоритм, функциональный вычислительный статистический алгоритм, условно-оптимальные параметры.

1. РЕШЕНИЕ ЗАДАЧИ ОПЕРАТИВНОГО ЧИСЛЕННОГО ФУНКЦИОНАЛЬНОГО ПРИБЛИЖЕНИЯ ВЕРОЯТНОСТНОЙ ПЛОТНОСТИ

В [1]–[4] мы рассмотрели следующую задачу.

Задача 1. По заданной выборке $\left\{ {{{\eta }_{1}},\; \ldots ,\;{{\eta }_{{\hat {n}}}}} \right\}$ построить численное функциональное приближение неизвестной плотности ${{f}_{\eta }}({\mathbf{x}})$, ${\mathbf{x}} \in X \subset {{\mathbb{R}}^{d}}$, случайной величины (вектора) $\eta \in X \subset {{\mathbb{R}}^{d}}$ с заданным уровнем погрешности $L > 0$ и с наименьшими вычислительными затратами $S$.

Сформулированная задача может быть весьма актуальной при оперативной обработке больших данных, т.е. при $\hat {n} \gg 1$ (см., например, [5], [6]).

Наше предложение состоит в том, чтобы, используя относительно небольшую часть $\left\{ {{{\eta }_{1}},\; \ldots ,\;{{\eta }_{n}}} \right\}$ выборки $\left\{ {{{\eta }_{1}},\; \ldots ,\;{{\eta }_{{\hat {n}}}}} \right\}$ (здесь $n \ll \hat {n}$), применить следующий функциональный вычислительный ядерный алгоритм.

Алгоритм 1 (см. [1]–[4]). Вычисляем значения

(1.1)
$\tilde {f}_{\eta }^{{({{{\mathbf{x}}}_{i}})}}(n) = {{Z}_{n}}({{{\mathbf{x}}}_{i}}) = \frac{1}{n}\sum\limits_{j = 1}^n {{{\kappa }^{{({{{\mathbf{x}}}_{i}})}}}} ({{\eta }_{{\mathbf{j}}}}),\quad i = 1,\;2,\; \ldots ,\;M,$
и приближаем функцию ${{f}_{\eta }}({\mathbf{x}})$ по формуле

(1.2)
${{f}_{\eta }}({\mathbf{x}}) \approx {{L}^{{(M,n)}}}{{\tilde {f}}_{\eta }}({\mathbf{x}}) = \sum\limits_{i = 1}^M {{{w}^{{(i)}}}} [\tilde {f}_{\eta }^{{({{{\mathbf{x}}}_{1}})}}(n),\; \ldots ,\;\tilde {f}_{\eta }^{{({{{\mathbf{x}}}_{M}})}}(n)]{{\chi }^{{(i)}}}({\mathbf{x}}).$

Величина ${{Z}_{n}}({{{\mathbf{x}}}_{i}})$ из соотношения (1.1) представляет собой ядерную оценку плотности ${{f}_{\eta }}({\mathbf{x}})$ (см. [7]) в точке ${{{\mathbf{x}}}_{i}}$, являющейся узлом аппроксимационной сетки

(1.3)
${{X}^{{(M)}}} = \{ {{{\mathbf{x}}}_{1}},\; \ldots ,\;{{{\mathbf{x}}}_{M}}\} ,$
введенной в области $X$. При этом ${{\kappa }^{{({\mathbf{x}})}}}({\mathbf{y}})$ – некоторая финитная параметрическая, одинаковая по форме для всех значений параметра ${\mathbf{x}}$, ядерная функция. Эта функция выбирается таким образом, чтобы выполнялись соотношения

(1.4)
${{f}_{\eta }}({{{\mathbf{x}}}_{i}}) \approx \tilde {f}_{\eta }^{{({{{\mathbf{x}}}_{i}})}}(n) = {{Z}_{n}}({{{\mathbf{x}}}_{i}}) = \frac{1}{n}\sum\limits_{j = 1}^n {{{\kappa }^{{({{{\mathbf{x}}}_{i}})}}}} ({{\eta }_{{\mathbf{j}}}}) \approx {\mathbf{E}}{{\kappa }^{{({{{\mathbf{x}}}_{i}})}}}\left( \eta \right) = \int {{{\kappa }^{{({{{\mathbf{x}}}_{i}})}}}} ({\mathbf{y}}){{f}_{\eta }}({\mathbf{y}})d{\mathbf{y}},\quad i = 1,\;2,\; \ldots ,\;M.$

В свою очередь, выражение (1.2) представляет собой классическое сеточное приближение на сетке (1.3) (см., например, ${{\S}}\,2$ гл. 2 [8], ${{\S}}\,2$, гл. 2) с аппроксимационным базисом

(1.5)
${{\Xi }^{{({\mathbf{M}})}}} = \{ {{\chi }^{{(1)}}}({\mathbf{x}}),\; \ldots ,\;{{\chi }^{{({\mathbf{M}})}}}({\mathbf{x}})\} $
(эти функции определенным образом связаны с сеткой (1.3)), а значения
$\mathop {{\mathbf{\tilde {W}}}}\nolimits^{(M)} (n) = \left\{ {{{w}^{{(1)}}}[\tilde {f}_{\eta }^{{({{{\mathbf{x}}}_{1}})}}(n),\; \ldots ,\;\tilde {f}_{\eta }^{{({{{\mathbf{x}}}_{M}})}}(n)],\; \ldots ,\;{{w}^{{(M)}}}[\tilde {f}_{\eta }^{{({{{\mathbf{x}}}_{1}})}}(n),\; \ldots ,\;\tilde {f}_{\eta }^{{({{{\mathbf{x}}}_{M}})}}(n)]} \right\}$
представляют собой аппроксимационные коэффициенты, являющиеся определенными комбинациями приближенных значений функции ${{f}_{\eta }}({\mathbf{x}})$ в узлах сетки (1.3) (см. соотношение (1.4)); чаще всего

(1.6)
${{w}^{{(i)}}}[\tilde {f}_{\eta }^{{({{{\mathbf{x}}}_{1}})}}(n),\; \ldots ,\;\tilde {f}_{\eta }^{{({{{\mathbf{x}}}_{M}})}}(n)] = \tilde {f}_{\eta }^{{({{{\mathbf{x}}}_{i}})}}(n),\quad i = 1,\;2,\; \ldots ,\;M.$

Помимо наших работ [1]–[4] идеология построения алгоритма 1 отражена в [9], [10], но необходимые детали использования теории сеточной аппроксимации функций в этих работах не приводятся.

Важным является следующее обстоятельство.

Замечание 1 (см. [1]–[4]). Конструкция функционального вычислительного ядерного алгоритма приближения вероятностной плотности ${{f}_{\eta }}({\mathbf{x}})$ близка к схеме построения функционального вычислительного ядерного статистического алгоритма приближения решения интегрального уравнения Фредгольма II рода (см. [11], разд. 8.6), и поэтому при исследовании алгоритма 1 целесообразно использовать соображения теории условной оптимизации функциональных алгоритмов метода Монте-Карло (см. прежде всего [12]–[21]).

Данная работа посвящена детализации соображений из замечания 1.

2. ОБЩАЯ СХЕМА УСЛОВНОЙ ОПТИМИЗАЦИИ

Общая схема оптимизации из [12]–[21] выглядит следующим образом. Ставится задача согласованного выбора параметров $M$ и $n$ (число узлов и выборочных значений из соотношений (1.1)) алгоритма 1, обеспечивающего заданный уровень $L > 0$ погрешности приближения (1.2) при минимальных вычислительных затратах $S(M,n)$.

Метод 1. Строится верхняя граница $U{{P}^{{(\mathbb{B})}}}(M,n)$ погрешности алгоритма ${{\delta }^{{(\mathbb{B})}}}(M,n)$ для используемого нормированного функционального пространства $\mathbb{B}(X)$, зависящая от параметров $M$ и $n$:

(2.1)
${{\delta }^{{(\mathbb{B})}}}(M,n) = \mathop {\left\| {{{f}_{\eta }} - {{L}^{{(M,n)}}}{{{\tilde {f}}}_{\eta }}} \right\|}\nolimits_{\mathbb{B}(X)} \leqslant U{{P}^{{(\mathbb{B})}}}(M,n).$

Эта функция двух переменных приравнивается величине $L$. Из уравнения вида

(2.2)
$U{{P}^{{(\mathbb{B})}}}(M,n) = L$
один из параметров (например, $n$) выражается через другой: $n = {{\psi }^{{(L)}}}(M)$. Это соотношение подставляется в выражение для затрат $S(M,n)$ (которое тоже зависит от параметров $M$ и $n$). В результате получается функция ${{\tilde {S}}^{{(\mathbb{B},L)}}}(M)$ одного переменного $M$, которая исследуется на минимум с помощью известных приемов математического или численного анализа. Найденные значения $M_{{{\text{min}}}}^{{(\mathbb{B})}}(L) = M_{{{\text{opt}}}}^{{(\mathbb{B})}}(L)$, $n_{{{\text{opt}}}}^{{(\mathbb{B})}}(L) = \psi [M_{{{\text{opt}}}}^{{(\mathbb{B})}}(L)]$ объявляются условно-оптимальными параметрами соответствующего функционального алгоритма.

“Условность” оптимизации по методу 1 связана с тем, что в левой части уравнения вида (2.2) используется не сама погрешность алгоритма ${{\delta }^{{(\mathbb{B})}}}(M,n)$, а ее верхняя граница $U{{P}^{{(\mathbb{B})}}}(M,n)$. К слову, оценка качества того или иного алгоритма по верхней границе погрешности используется в подавляющем числе теоретических рассуждений вычислительной математики (см., например, [8]), т.е. по сути речь идет об “условной оптимальности” исследуемых численных схем.

При изучении погрешности ${{\delta }^{{(\mathbb{B})}}}(M,n)$ необходимо выбрать как соответствующее нормированное функциональное пространство $\mathbb{B}(X)$, так и вероятностный смысл выполнения неравенства вида (2.1) (ведь ${{\delta }^{{(\mathbb{B})}}}(M,n)$ является случайной величиной). Следуя канонам классического численного анализа (см., например, [8]), в качестве пространств $\mathbb{B}(X)$ будем рассматривать ${{\mathbb{L}}_{2}}(X)$ и $\mathbb{C}(X)$.

Для хорошо разработанного (см. в первую очередь [12]–[21]) ${{\mathbb{L}}_{2}}$-подхода выбирается сходимость в среднем погрешности

${{\delta }^{{({{\mathbb{L}}_{2}})}}}(M,n) = \mathop {\left\| {{{f}_{\eta }} - {{L}^{{(M,n)}}}{{{\tilde {f}}}_{\eta }}} \right\|}\nolimits_{{{\mathbb{L}}_{2}}(X)} = {{\left( {\int\limits_X {{{{[{{f}_{\eta }}({\mathbf{x}}) - {{L}^{{(M,n)}}}{{{\tilde {f}}}_{\eta }}({\mathbf{x}})]}}^{2}}} d{\mathbf{x}}} \right)}^{{1/2}}}$
к нулю при $M,n \to \infty $ и строятся оценки сверху $U{{P}^{{({{\mathbb{L}}_{2}})}}}(M,n)$ такие, что

(2.3)
${{[{\mathbf{E}}{{\delta }^{{({{\mathbb{L}}_{2}})}}}(M,n)]}^{2}} \leqslant U{{P}^{{({{\mathbb{L}}_{2}})}}}(M,n).$

Для $\mathbb{C}$-подхода (см. [16]–[21]) величина

${{\delta }^{{(\mathbb{C})}}}(M,n) = \mathop {\left\| {{{f}_{\eta }} - {{L}^{{(M,n)}}}{{{\tilde {f}}}_{\eta }}} \right\|}\nolimits_{\mathbb{C}(X)} = \mathop {sup}\limits_{{\mathbf{x}} \in X} \left| {{{f}_{\eta }}({\mathbf{x}}) - {{L}^{{(M,n)}}}{{{\tilde {f}}}_{\eta }}({\mathbf{x}})} \right|$
ограничивается сверху по вероятности
(2.4)
${\mathbf{P}}[{{\delta }^{{(\mathbb{C})}}}(M,n) \leqslant U{{P}^{{(\mathbb{C})}}}(M,n)] > 1 - \varepsilon $
для некоторого достаточно малого $\varepsilon > 0$.

Отметим, что для ${{\mathbb{L}}_{2}}$-подхода используется относительно “слабая” интегральная норма ${{\left\| {\, \cdot \,} \right\|}_{{{{\mathbb{L}}_{2}}(X)}}}$ пространства ${{\mathbb{L}}_{2}}(X)$ и “сильная” вероятностная сходимость погрешности к нулю (в среднем). В свою очередь в $\mathbb{C}$-подходе для “жесткой” нормы ${{\left\| {\, \cdot \,} \right\|}_{{\mathbb{C}(X)}}}$ используется относительно “слабая” сходимость (по вероятности).

Для $\mathbb{C}$- и ${{\mathbb{L}}_{2}}$-подходов можно разделить погрешность на три компоненты: детерминированную $\delta _{{{\text{det}}}}^{{(\mathbb{B})}}(M)$, стохастическую $\delta _{{{\text{stoch}}}}^{{(\mathbb{B})}}(M,n)$ и компоненту смещения $\delta _{{{\text{bias}}}}^{{(\mathbb{B})}}(M)$.

В частности, для $\mathbb{C}$-подхода, согласно неравенству треугольника,

(2.5)
$\begin{gathered} {{\delta }^{{(\mathbb{C})}}}(M,n) \leqslant \delta _{{{\text{det}}}}^{{(\mathbb{C})}}(M) + \delta _{{{\text{stoch}}}}^{{(\mathbb{C})}}(M,n) + \delta _{{{\text{bias}}}}^{{(\mathbb{C})}}(M),\quad \delta _{{{\text{det}}}}^{{(\mathbb{C})}}(M) = \mathop {\left\| {{{f}_{\eta }} - {{L}^{{(M)}}}{{f}_{\eta }}} \right\|}\nolimits_{\mathbb{C}(X)} , \\ \delta _{{{\text{stoch}}}}^{{(\mathbb{C})}}(M,n) = \mathop {\left\| {{{L}^{{(M)}}}{{{\bar {f}}}_{\eta }} - {{L}^{{(M,n)}}}{{{\tilde {f}}}_{\eta }}} \right\|}\nolimits_{\mathbb{C}(X)} ,\quad \delta _{{{\text{bias}}}}^{{(\mathbb{C})}}(M) = \mathop {\left\| {{{L}^{{(M)}}}{{f}_{\eta }} - {{L}^{{(M)}}}{{{\bar {f}}}_{\eta }}} \right\|}\nolimits_{\mathbb{C}(X)} . \\ \end{gathered} $
Здесь

(2.6)
${{L}^{{(M)}}}{{f}_{\eta }}({\mathbf{x}}) = \sum\limits_{i = 1}^M {{{w}^{{(i)}}}} [{{f}_{\eta }}({{{\mathbf{x}}}_{1}}),\; \ldots ,\;{{f}_{\eta }}({{{\mathbf{x}}}_{M}})]{{\chi }^{{(i)}}}({\mathbf{x}}),$
(2.7)
${{L}^{{(M)}}}{{\bar {f}}_{\eta }}({\mathbf{x}}) = \sum\limits_{i = 1}^M {{{w}^{{(i)}}}} [{\mathbf{E}}{{\kappa }^{{({{{\mathbf{x}}}_{1}})}}}\left( \eta \right),\; \ldots ,\;{\mathbf{E}}{{\kappa }^{{({{{\mathbf{x}}}_{M}})}}}\left( \eta \right)]{{\chi }^{{(i)}}}({\mathbf{x}}).$

Для ${{\mathbb{L}}_{2}}$-подхода, применяя неравенство Коши–Буняковского и теорему Фубини, получаем

${{[{\mathbf{E}}{{\delta }^{{({{\mathbb{L}}_{2}})}}}(M,n)]}^{2}} \leqslant {\mathbf{E}}\left( {\int\limits_X {{{{[{{f}_{\eta }}({\mathbf{x}}) - {{L}^{{(M,n)}}}{{{\tilde {f}}}_{\eta }}({\mathbf{x}})]}}^{2}}d{\mathbf{x}}} } \right) \times {\mathbf{E}}1 = \int\limits_X {\mathbf{E}} {{[{{f}_{\eta }}({\mathbf{x}}) - {{L}^{{(M,n)}}}{{\tilde {f}}_{\eta }}({\mathbf{x}})]}^{2}}d{\mathbf{x}}.$
Далее заметим, что
$\begin{gathered} {\mathbf{E}}{{[{{f}_{\eta }}({\mathbf{x}}) - {{L}^{{(M,n)}}}{{{\tilde {f}}}_{\eta }}({\mathbf{x}})({\mathbf{x}})]}^{2}} = {\mathbf{E}}{{\left( {[{{f}_{\eta }}({\mathbf{x}}) - {{L}^{{(M)}}}{{f}_{\eta }}({\mathbf{x}})] + [{{L}^{{(M)}}}{{f}_{\eta }}({\mathbf{x}}) - {{L}^{{(M)}}}{{{\bar {f}}}_{\eta }}({\mathbf{x}})] + [{{L}^{{(M)}}}{{{\bar {f}}}_{\eta }}({\mathbf{x}}) - {{L}^{{(M,n)}}}{{{\tilde {f}}}_{\eta }}({\mathbf{x}})]} \right)}^{2}} = \\ = \mathop {\left( {[{{f}_{\eta }}({\mathbf{x}}) - {{L}^{{(M)}}}{{f}_{\eta }}({\mathbf{x}})] + [{{L}^{{(M)}}}{{f}_{\eta }}({\mathbf{x}}) - {{L}^{{(M)}}}{{{\bar {f}}}_{\eta }}({\mathbf{x}})]} \right)}\nolimits^2 + {\mathbf{E}}{{[{{L}^{{(M)}}}{{{\bar {f}}}_{\eta }}({\mathbf{x}}) - {{L}^{{(M,n)}}}{{{\tilde {f}}}_{\eta }}({\mathbf{x}})]}^{2}}. \\ \end{gathered} $
Здесь учтено, что ${\mathbf{E}}{{L}^{{(M,n)}}}{{\tilde {f}}_{\eta }}({\mathbf{x}}) = {{L}^{{(M)}}}{{\bar {f}}_{\eta }}({\mathbf{x}})$ (см. соотношения (1.2), (1.4), (2.7)). Используя очевидное неравенство ${{(a + b)}^{2}} \leqslant 2{{a}^{2}} + 2{{b}^{2}}$, получаем

(2.8)
$\begin{gathered} {{[{\mathbf{E}}{{\delta }^{{({{\mathbb{L}}_{2}})}}}(M,n)]}^{2}} \leqslant 2{{[\delta _{{{\text{det}}}}^{{({{\mathbb{L}}_{2}})}}(M)]}^{2}} + \delta _{{{\text{stoch}}}}^{{({{\mathbb{L}}_{2}})}}(M,n) + 2{{[\delta _{{{\text{bias}}}}^{{({{\mathbb{L}}_{2}})}}(M)]}^{2}},\quad \delta _{{{\text{det}}}}^{{({{\mathbb{L}}_{2}})}}(M) = {{\left\| {{{f}_{\eta }} - {{L}^{{(M)}}}{{f}_{\eta }}} \right\|}_{{{{\mathbb{L}}_{2}}(X)}}}, \\ \delta _{{{\text{stoch}}}}^{{({{\mathbb{L}}_{2}})}}(M,n) = \int\limits_X {\mathbf{D}} {{L}^{{(M,n)}}}{{{\tilde {f}}}_{\eta }}({\mathbf{x}})d{\mathbf{x}},\quad \delta _{{{\text{bias}}}}^{{({{\mathbb{L}}_{2}})}}(M) = \mathop {\left\| {{{L}^{{(M)}}}{{f}_{\eta }} - {{L}^{{(M)}}}{{{\bar {f}}}_{\eta }}} \right\|}\nolimits_{{{\mathbb{L}}_{2}}(X)} . \\ \end{gathered} $

3. ВЕРХНИЕ ГРАНИЦЫ ДЛЯ ДЕТЕРМИНИРОВАННЫХ КОМПОНЕНТ $\mathbb{C}$- И ${{\mathbb{L}}_{2}}$-ПОГРЕШНОСТЕЙ

В книгах и диссертациях [14], [16]–[21] и сопутствующих статьях [22]–[24] по теории условной оптимизации по методу 1 приведены весомые аргументы в пользу использования в качестве функций (1.5) базиса Стренга–Фикса (см. [25])

(3.1)
${{\chi }^{{(i,r)}}}({\mathbf{x}}) = {{\beta }^{{(r)}}}\left( {\frac{{{{x}^{{(1)}}}}}{h} - j_{i}^{{(1)}}} \right) \times \; \ldots \; \times {{\beta }^{{(r)}}}\left( {\frac{{{{x}^{{(d)}}}}}{h} - j_{i}^{{(d)}}} \right)$
для равномерной сетки (1.3) с шагом $h$ по каждой координате, т.е. с узлами вида ${{{\mathbf{x}}}_{i}} = (j_{i}^{{(1)}}h,\; \ldots ,\;j_{i}^{{(d)}}h)$ в прямоугольной области $X$; здесь $\{ j_{i}^{{(k)}},i = 1,\;2,\; \ldots ,\;M,\;k = 1,\;2,\; \ldots ,\;d\} $ – целые числа.

Для такой сетки число $M$ узлов сетки пропорционально величине ${{h}^{{ - d}}}$, т.е. существует константа ${{H}^{{(h \to M)}}}$ такая, что

(3.2)
$h = {{H}^{{(h \to M)}}}{{M}^{{ - 1/d}}}.$

В формуле (3.1) функция ${{\beta }^{{(r)}}}(u)$ представляет собой $B$-сплайн $r$-го порядка (см., например, [25]), который можно определить с помощью следующих рекуррентных формул:

(3.3)
${{\beta }^{{(r)}}}(u) = {{\beta }^{{(r - 1)}}} * {{\beta }^{{(0)}}}(u)\mathop = \limits^{{\text{def}}} \int {{{\beta }^{{(r - 1)}}}} ({v}){{\beta }^{{(0)}}}(u - {v})d{v},\quad {{\beta }^{{(0)}}}(u) = \left\{ {\begin{array}{*{20}{c}} {1\quad {\text{при}}\quad \left| u \right| \leqslant 1{\text{/}}2,} \\ {0\quad {\text{при}}\quad \left| u \right| > 1{\text{/}}2.} \end{array}} \right.$

Учитывая вид линейных преобразований координат $\{ {{x}^{{(1)}}},\; \ldots ,\;{{x}^{{(d)}}}\} $ в формуле (3.1), важным является требование того, что сплайн ${{\beta }^{{(r)}}}(u)$ должен представлять собой кусочно-гладкую функцию, причем границы разбиения интервала, на котором ${{\beta }^{{(r)}}}(u) > 0$, на соответствующие полуинтервалы должны быть целыми точками. Это требование выполняется, когда параметр $r$ является натуральным нечетным числом: $r = 1,\;3,\;5,\; \ldots $ . Чаще всего используется $B$-сплайн первого порядка (или “функция-крышка”)

(3.4)
${{\beta }^{{(1)}}}(u) = \left\{ {\begin{array}{*{20}{l}} {u + 1\quad {\text{при}}\quad - 1 \leqslant u \leqslant 0,} \\ { - u + 1\quad {\text{при}}\quad 0 \leqslant u \leqslant 1,} \\ {0\quad {\text{при}}\quad \left| u \right| > 1.} \end{array}} \right.$
При этом приближение (2.3) с базисными функциями $\{ {{\chi }^{{(i,1)}}}({\mathbf{x}});\;i = 1,\;2,\; \ldots ,\;M\} $ и с коэффициентами
(3.5)
${{W}^{{(M)}}}({{{\mathbf{f}}}_{\eta }}) = \left\{ {{{w}^{{(1)}}}[{{f}_{\eta }}({{{\mathbf{x}}}_{1}}),\; \ldots ,\;{{f}_{\eta }}({{{\mathbf{x}}}_{M}})],\; \ldots ,\;{{w}^{{(M)}}}[{{f}_{\eta }}({{{\mathbf{x}}}_{1}}),\; \ldots ,\;{{f}_{\eta }}({{{\mathbf{x}}}_{M}})]} \right\}$
простейшего вида
(3.6)
${{w}^{{(i)}}}[{{f}_{\eta }}({{{\mathbf{x}}}_{1}}),\; \ldots ,\;{{f}_{\eta }}({{{\mathbf{x}}}_{M}})] = {{f}_{\eta }}({{{\mathbf{x}}}_{i}}),\quad i = 1,\;2,\; \ldots ,\;M$
(это аналог соотношения (1.6)) определяет мультилинейную аппроксимацию плотности ${{f}_{\eta }}({\mathbf{x}})$ (см. [14], [16]–[21]).

На основании подходов из [20], [25] можно получить следующие результаты.

Утверждение 1. Если функция ${{f}_{\eta }}({\mathbf{x}})$ принадлежит пространству ${{\mathbb{C}}^{{r + 1}}}(X)$ и в приближении (2.6) функции ${{f}_{\eta }}({\mathbf{x}})$ используются базисные функции $\{ {{\chi }^{{(i,r)}}}({\mathbf{x}});\;i = 1,\;2,\; \ldots ,\;M\} $ из (3.1), то найдутся такие коэффициенты (3.5), что справедливы неравенства

${{\left\| {{{f}_{\eta }} - {{L}^{{(M)}}}{{f}_{\eta }}} \right\|}_{{{{\mathbb{C}}^{s}}(X)}}} \leqslant \tilde {H}_{r}^{{({{\mathbb{C}}^{s}})}}{{h}^{{r + 1 - s}}}{{\left\| {{{f}_{\eta }}} \right\|}_{{{{\mathbb{C}}^{{r + 1}}}(X)}}},\quad 0 \leqslant s \leqslant r,$
причем константы $\tilde {H}_{r}^{{({{\mathbb{C}}^{s}})}}$ не зависят от ${{f}_{\eta }}({\mathbf{x}})$ и $h$.

Утверждение 2. Если функция ${{f}_{\eta }}({\mathbf{x}})$ принадлежит пространству $\mathbb{W}_{2}^{{r + 1}}(X)$ и в приближении (2.6) используются базисные функции (3.1), то найдутся такие коэффициенты (3.5), что справедливы неравенства

${{\left\| {{{f}_{\eta }} - {{L}^{{(M)}}}{{f}_{\eta }}} \right\|}_{{\mathbb{W}_{2}^{s}(X)}}} \leqslant \tilde {H}_{r}^{{(\mathbb{W}_{2}^{s})}}{{h}^{{r + 1 - s}}}{{\left\| {{{f}_{\eta }}} \right\|}_{{\mathbb{W}_{2}^{{r + 1}}(X)}}},\quad 0 \leqslant s \leqslant r,$
причем константы $\tilde {H}_{r}^{{(\mathbb{W}_{2}^{s})}}$ не зависят от ${{f}_{\eta }}({\mathbf{x}})$ и $h$.

Заметим, что

$\delta _{{{\text{det}}}}^{{(\mathbb{C})}}(M) = {{\left\| {{{f}_{\eta }} - {{L}^{{(M)}}}{{f}_{\eta }}} \right\|}_{{{{\mathbb{C}}^{0}}(X)}}}\quad {\text{и}}\quad \delta _{{{\text{det}}}}^{{({{\mathbb{L}}_{2}})}}(M) = {{\left\| {{{f}_{\eta }} - {{L}^{{(M)}}}{{f}_{\eta }}} \right\|}_{{\mathbb{W}_{2}^{0}(X)}}}.$

Из утверждений 1 и 2 и соотношения (3.2) получаем

(3.7)
$\begin{gathered} \delta _{{{\text{det}}}}^{{(\mathbb{C})}}(M) \leqslant \tilde {H}_{r}^{{(\mathbb{C})}}{{[{{H}^{{(h \to M)}}}]}^{{r + 1}}}{{M}^{{ - (r + 1)/d}}}{{\left\| {{{f}_{\eta }}} \right\|}_{{{{\mathbb{C}}^{{r + 1}}}(X)}}}, \\ \delta _{{{\text{det}}}}^{{({{\mathbb{L}}_{2}})}}(M) \leqslant \tilde {H}_{r}^{{({{\mathbb{L}}_{2}})}}{{[{{H}^{{(h \to M)}}}]}^{{r + 1}}}{{M}^{{ - (r + 1)/d}}}{{\left\| {{{f}_{\eta }}} \right\|}_{{\mathbb{W}_{2}^{{r + 1}}(X)}}}. \\ \end{gathered} $

Особую проблему представляет собой нахождение коэффициентов (3.5), обеспечивающих выполнение неравенств вида (3.7). Относительно несложным является лишь случай $r = 1$.

Теорема 1 (см. [20], [25]). Выбор базисных функций (3.1), (3.4) и коэффициентов (3.6) мультилинейной аппроксимации обеспечивает выполнение неравенств вида (3.7)

(3.8)
$\delta _{{{\text{det}}}}^{{(\mathbb{C})}}(M) \leqslant H_{{{\text{det}}}}^{{(\mathbb{C})}}{{M}^{{ - 2/d}}},\quad \delta _{{{\text{det}}}}^{{({{\mathbb{L}}_{2}})}}(M) \leqslant H_{{{\text{det}}}}^{{({{\mathbb{L}}_{2}})}}{{M}^{{ - 2/d}}}$
при ${{f}_{\eta }} \in {{\mathbb{C}}^{2}}(X)$ и ${{f}_{\eta }} \in \mathbb{W}_{2}^{2}(X)$ соответственно. При этом

(3.9)
$H_{{{\text{det}}}}^{{(\mathbb{C})}} = \frac{{{{{[{{H}^{{(h \to M)}}}]}}^{2}}}}{8}\sum\limits_{s = 1}^d {\mathop {max}\limits_{{\mathbf{x}} \in X} } \left| {\frac{{{{\partial }^{2}}}}{{\partial {{{({{x}^{{(s)}}})}}^{2}}}}{{f}_{\eta }}({\mathbf{x}})} \right|\quad и\quad H_{{{\text{det}}}}^{{({{\mathbb{L}}_{2}})}} = H_{{{\text{det}}}}^{{(\mathbb{C})}} \times \sqrt {\operatorname{mes} X} .$

В случае $r > 1$ выбор подходящих коэффициентов (3.5), обеспечивающих выполнение неравенств вида (3.7), существенно более сложен. Отметим, что в [20] получены коэффициенты (3.5), представляющие собой специальные (существенно отличные от (3.6)) комбинации значений $\{ {{f}_{\eta }}({{{\mathbf{x}}}_{1}}),\; \ldots ,\;{{f}_{\eta }}({{{\mathbf{x}}}_{M}})\} $ и обеспечивающие оптимальный порядок ${{M}^{{ - 4/d}}}$ (см. утверждения 1, 2 и соотношение (3.2)) величин $\delta _{{{\text{det}}}}^{{(\mathbb{C})}}(M)$, $\delta _{{{\text{det}}}}^{{({{\mathbb{L}}_{2}})}}(M)$ для ${{f}_{\eta }} \in {{\mathbb{C}}^{4}}(X)$ и ${{f}_{\eta }} \in \mathbb{W}_{2}^{4}(X)$, соответственно, и для функций (3.1) при $r = 3$. Здесь

(3.10)
${{\beta }^{{(3)}}}(u) = \left\{ \begin{gathered} 0\quad {\text{при}}\quad u > 2, \hfill \\ {{(2 - u)}^{3}}{\text{/}}6\quad {\text{при}}\quad 1 \leqslant u \leqslant 2, \hfill \\ [1 + 3(1 - u) + 3{{(1 - u)}^{2}} - 3{{(1 - u)}^{3}}]{\text{/}}6\quad {\text{при}}\quad 0 \leqslant u \leqslant 1, \hfill \\ {{\beta }^{{(3)}}}( - u)\quad {\text{при}}\quad u \leqslant 0. \hfill \\ \end{gathered} \right.$

Дальнейшие рассуждения показывают, что для исследуемого алгоритма 1 достаточно (и даже необходимо) рассмотрение случая $r = 1$ (т.е. использование мультилинейного восполнения (3.1), (3.4)) и выполнение принципиальных соотношений (3.6) и (3.8). В частности, это обусловлено тем, что компоненты смещения $\delta _{{{\text{bias}}}}^{{(\mathbb{C})}}(M)$, $\delta _{{{\text{bias}}}}^{{({{\mathbb{L}}_{2}})}}(M)$ имеют порядок ${{M}^{{ - 2/d}}}$ (как и детерминированные компоненты из соотношений (3.8)), во всяком случае, для простейшей “ядерной” функции (4.5) (см. далее разд. 4).

4. ВЕРХНИЕ ГРАНИЦЫ ДЛЯ КОМПОНЕНТ СМЕЩЕНИЯ $\mathbb{C}$- И ${{\mathbb{L}}_{2}}$-ПОГРЕШНОСТЕЙ

Для обоснования последнего замечания из предыдущего раздела нам понадобится следующий факт.

Утверждение 3. Базис Стренга–Фикса (3.1) является разложением единицы, т.е. $\sum\nolimits_i {{{\chi }^{{(i,r)}}}} ({\mathbf{x}}) \equiv 1$ для всех ${\mathbf{x}} \in {{\mathbb{R}}^{d}}$ и $r \in \mathbb{N}$.

Доказательство. Используем метод математической индукции по размерности $d$ пространства ${{\mathbb{R}}^{d}}$.

Для $d = 1$ утверждение доказываем индукцией по порядку $r$ сплайна ${{\beta }^{{(r)}}}(u)$. Пусть $r = 0$ и $x \in \mathbb{R}$. Найдется единственное целое число $\hat {i}$ такое, что $\hat {i}h - h{\text{/}}2 \leqslant x < \hat {i}h + h{\text{/}}2$. Тогда $\sum\nolimits_i {{{\beta }^{{(0)}}}} (x{\text{/}}h - i) \equiv 1$ (см. соотношения (3.3)), так как в этой сумме все слагаемые равны нулю, кроме $\hat {i}$-го, которое равно единице.

Пусть теперь

(4.1)
$\sum\limits_i {{{\beta }^{{(r - 1)}}}} \left( {\frac{x}{h} - i} \right) \equiv 1$
для всех $x \in \mathbb{R}$. Тогда, согласно определению $B$-сплайна (см. соотношения (3.3)), имеем
$\begin{gathered} \sum\limits_i {{{\beta }^{{(r)}}}} \left( {\frac{x}{h} - i} \right) = \sum\limits_i {\int\limits_{ - \infty }^{ + \infty } {{{\beta }^{{(0)}}}} } (y){{\beta }^{{(r - 1)}}}\left( {\frac{{x - ih}}{h} - y} \right)dy = \\ = \;\int\limits_{ - \infty }^{ + \infty } {{{\beta }^{{(0)}}}} (y)\sum\limits_i {{{\beta }^{{(r - 1)}}}} \left( {\frac{{x - ih}}{h} - y} \right)dy = \int\limits_{ - 1/2}^{ + 1/2} {\sum\limits_i {{{\beta }^{{(r - 1)}}}} \left[ {\frac{{(x - hy) - ih}}{h}} \right]dy} . \\ \end{gathered} $
Используя индуктивное предположение (5.1) для $(x - hy)$ вместо $x$, получаем $\sum\nolimits_i {{{\beta }^{{(r)}}}} (x{\text{/}}h - i) \equiv 1$, т.е. утверждение 3 верно для $d = 1$.

Наконец, индуктивный переход по размерности $d$ следует из соотношения

$\sum\limits_i {{{\chi }^{{(i,r)}}}} ({\mathbf{x}}) = \sum\limits_{j_{{(i)}}^{{(1)}}, \ldots ,j_{{(i)}}^{{(d)}}} {\chi _{{(j_{{(i)}}^{{(1)}}, \ldots ,j_{{(i)}}^{{(d)}})}}^{{(r)}}} ({{x}^{{(1)}}},\; \ldots ,\;{{x}^{{(d)}}}) = \sum\limits_{j_{{(i)}}^{{(1)}}, \ldots ,j_{{(i)}}^{{(d - 1)}}} {\chi _{{(j_{{(i)}}^{{(1)}}, \ldots ,j_{{(i)}}^{{(d - 1)}})}}^{{(r)}}} ({{x}^{{(1)}}},\; \ldots ,\;{{x}^{{(d - 1)}}}) \times \sum\limits_{j_{{(i)}}^{{(d)}}} {{{\beta }^{{(r)}}}} \left( {\frac{{{{x}^{{(d)}}}}}{h} - j_{{(i)}}^{{(d)}}} \right).$
Утверждение 3 доказано.

Из утверждения 3 следует, что константа Лебега ${{K}^{{({{\Xi }^{{(M)}}})}}}$ (см. [26, разд. 3.1]) для базиса Стренга–Фикса ${{\Xi }^{{(M)}}}$ вида (3.1) равна единице:

${{K}^{{({{\Xi }^{{({\mathbf{M}})}}})}}}\mathop = \limits^{{\text{def}}} \mathop {sup}\limits_{{\mathbf{x}} \in X} \sum\limits_{i = 1}^M {\left| {{{\chi }^{{(i,r)}}}({\mathbf{x}})} \right|} = \mathop {sup}\limits_{{\mathbf{x}} \in X} \sum\limits_{i = 1}^M {{{\chi }^{{(i,r)}}}} ({\mathbf{x}}) = 1,$
и в случае, когда в алгоритме 1 используются базис Стренга–Фикса (3.1) и коэффициенты (3.5) простейшего вида (3.6), выполнены соотношения

(4.2)
$\begin{gathered} \delta _{{{\text{bias}}}}^{{(\mathbb{C})}}(M) = \mathop {sup}\limits_{{\mathbf{x}} \in X} \left| {{{L}^{{(M)}}}{{f}_{\eta }}({\mathbf{x}}) - {{L}^{{(M)}}}{{{\bar {f}}}_{n}}({\mathbf{x}})} \right| = \mathop {sup}\limits_{{\mathbf{x}} \in X} \left| {\sum\limits_{i = 1}^M {{{f}_{\eta }}} ({{{\mathbf{x}}}_{i}}){{\chi }^{{(i)}}}({\mathbf{x}}) - \sum\limits_{i = 1}^M {{\mathbf{E}}{{\kappa }^{{({{{\mathbf{x}}}_{i}})}}}} (\eta ){{\chi }^{{(i)}}}({\mathbf{x}})} \right| \leqslant \\ \leqslant \;{{K}^{{({{\Xi }^{{(M)}}})}}} \times \mathop {max}\limits_{i = 1,...,M} \left| {{{f}_{\eta }}({{{\mathbf{x}}}_{i}}) - {\mathbf{E}}{{\kappa }^{{({{{\mathbf{x}}}_{i}})}}}(\eta )} \right| = \mathop {max}\limits_{i = 1,...,M} \left| {{{f}_{\eta }}({{{\mathbf{x}}}_{i}}) - \int {{{\kappa }^{{({{{\mathbf{x}}}_{i}})}}}} ({\mathbf{y}}){{f}_{\eta }}({\mathbf{y}})d{\mathbf{y}}} \right|, \\ \end{gathered} $
(4.3)
$\begin{gathered} \delta _{{{\text{stoch}}}}^{{(\mathbb{C})}}(M,n) = \mathop {sup}\limits_{{\mathbf{x}} \in X} \left| {{{L}^{{(M)}}}{{{\bar {f}}}_{\eta }}({\mathbf{x}}) - {{L}^{{(M,n)}}}{{{\tilde {f}}}_{\eta }}({\mathbf{x}})} \right| = \mathop {sup}\limits_{{\mathbf{x}} \in X} \left| {\int\limits_{i = 1}^M {{\mathbf{E}}{{\kappa }^{{({{{\mathbf{x}}}_{i}})}}}} (\eta ){{\chi }^{{(i)}}}({\mathbf{x}}) - \sum\limits_{i = 1}^M {\bar {f}_{\eta }^{{({{{\mathbf{x}}}_{i}})}}} (n){{\chi }^{{(i)}}}({\mathbf{x}})} \right| \leqslant \\ \leqslant \;{{K}^{{({{\Xi }^{{(M)}}})}}} \times \mathop {max}\limits_{i = 1,...,M} \left| {{\mathbf{E}}{{\kappa }^{{({{{\mathbf{x}}}_{i}})}}}(\eta ) - \tilde {f}_{\eta }^{{({{{\mathbf{x}}}_{i}})}}(n)} \right| = \mathop {max}\limits_{i = 1,...,M} \left| {{{Z}_{n}}({{{\mathbf{x}}}_{i}}) - {\mathbf{E}}{{\kappa }^{{({{{\mathbf{x}}}_{i}})}}}(\eta )} \right|. \\ \end{gathered} $

Заметим, что в [20] для аппроксимации Стренга–Фикса (2.6), (3.1) с кубической производящей функцией (3.10) для специального вида коэффициентов (3.5), дающих, согласно утверждениям 1 и 2, оптимальный порядок ${{M}^{{ - 4/d}}}$ величин $\delta _{{{\text{det}}}}^{{(\mathbb{C})}}(M)$, $\delta _{{{\text{det}}}}^{{({{\mathbb{L}}_{2}})}}(M)$, получено ${{K}^{{({{\Xi }^{{(M)}}})}}} = 3$ (что тоже неплохо). А вот, например, для одномерной интерполяции Лагранжа константа Лебега ${{K}^{{({{\Xi }^{{(M)}}})}}}$ растет с увеличением числа $M$ узлов равномерной сетки как ${{2}^{M}}$ (см. [26]).

Предположим, что $I$ является номером узла ${{{\mathbf{x}}}_{I}}$, в котором достигается максимум $ma{{x}_{{i = 1,...,M}}}\left| {{{f}_{\eta }}({{{\mathbf{x}}}_{i}}) - {\mathbf{E}}{{\kappa }^{{({{{\mathbf{x}}}_{i}})}}}\left( \eta \right)} \right|$ в правой части соотношения (4.2). Тогда

(4.4)
$\delta _{{{\text{bias}}}}^{{(\mathbb{C})}}(M) \leqslant \left| {{{f}_{\eta }}({{{\mathbf{x}}}_{I}}) - \int {{{\kappa }^{{({{{\mathbf{x}}}_{I}})}}}({\mathbf{y}}){{f}_{\eta }}({\mathbf{y}})d{\mathbf{y}}} } \right|.$

Далее последуем рекомендациям из [14], [16]–[23] и выберем кусочно-постоянный вариант ядерной функции

(4.5)
${{\kappa }^{{({\mathbf{x}})}}}({\mathbf{y}}) = \left\{ \begin{gathered} \frac{1}{{{{h}^{d}}}}\quad {\text{при}}\quad {\mathbf{y}} \in {{\Delta }^{{({\mathbf{x}})}}}, \hfill \\ 0\quad {\text{иначе}}. \hfill \\ \end{gathered} \right.$
Здесь
${{\Delta }^{{({\mathbf{x}})}}} = \left\{ {{\mathbf{y}} = ({{y}^{{(1)}}},\; \ldots ,\;{{y}^{{(d)}}}):{{x}^{{(s)}}} - \frac{h}{2} \leqslant {{y}^{{(s)}}} \leqslant {{x}^{{(s)}}} - \frac{h}{2};\;s = 1,\; \ldots ,\;d;\;{\mathbf{x}} = ({{x}^{{(1)}}},\; \ldots ,\;{{x}^{{(d)}}})} \right\}$
и ${\mathbf{x}} = ({{x}^{{(1)}}},\; \ldots ,\;{{x}^{{(d)}}})$. В этом случае имеем

(4.6)
$\left| {{{f}_{\eta }}({{{\mathbf{x}}}_{I}}) - \int {{{\kappa }^{{({{{\mathbf{x}}}_{I}})}}}} ({\mathbf{y}}){{f}_{\eta }}({\mathbf{y}})d{\mathbf{y}}} \right| = \frac{{\left| {{{h}^{d}}{{f}_{\eta }}({{{\mathbf{x}}}_{I}}) - \int\limits_{{{\Delta }^{{({{{\mathbf{x}}}_{I}})}}}} {{{f}_{\eta }}} ({\mathbf{y}})d{\mathbf{y}}ht} \right|}}{{{{h}^{d}}}}.$

Далее проводим следующие рассуждения из теории численного интегрирования, касающиеся обоснования квадратурной формулы центральных прямоугольников (см., например, [8, ${{\S}}\;1$ гл. 3]).

Утверждение 4. Если ${{f}_{\eta }} \in {{\mathbb{C}}^{2}}(X)$, то справедливо неравенство

(4.7)
${{J}^{{({{{\mathbf{x}}}_{I}})}}} = \frac{{\left| {{{h}^{d}}{{f}_{\eta }}({{{\mathbf{x}}}_{I}}) - \int\limits_{{{\Delta }^{{({{{\mathbf{x}}}_{I}})}}}} {{{f}_{\eta }}} ({\mathbf{y}})d{\mathbf{y}}} \right|}}{{{{h}^{d}}}} \leqslant \frac{{{{h}^{2}}}}{{24}}\sum\limits_{s = 1}^d {\mathop {max}\limits_{{\mathbf{y}} \in {{\Delta }^{{({{{\mathbf{x}}}_{I}})}}}} } \left| {\frac{{{{\partial }^{2}}{{f}_{\eta }}({\mathbf{y}})}}{{\partial {{{({{y}^{{(s)}}})}}^{2}}}}} \right|.$
Здесь ${\mathbf{y}} = ({{y}^{{(1)}}},\; \ldots ,\;{{y}^{{(M)}}})$.

Доказательство. Рассмотрим сначала случай $d = 1$. Здесь ${{\Delta }^{{({{x}_{I}})}}} = {\text{\{ }}y:$ ${{x}_{I}} - h{\text{/}}2 \leqslant y \leqslant {{x}_{I}} + h{\text{/}}2{\text{\} }}$. Несложно показать, что

$h{{f}_{\eta }}({{x}_{I}}) = \int\limits_{{{\Delta }^{{({{x}_{I}})}}}} {[{{f}_{\eta }}({{x}_{I}}) + f_{\eta }^{'}({{x}_{I}})(y - {{x}_{I}})]{\kern 1pt} dy} .$

Кроме того, согласно формуле Тейлора, имеем

${{f}_{\eta }}(y) = {{f}_{\eta }}({{x}_{I}}) + f_{\eta }^{'}({{x}_{I}})(y - {{x}_{I}}) + {{D}_{1}},$
где $y \in {{\Delta }^{{({{x}_{I}})}}}$ и ${{D}_{1}} \leqslant \tfrac{{{{{(y - {{x}_{I}})}}^{2}}}}{2}\mathop {max}\limits_{y \in {{\Delta }^{{({{x}_{I}})}}}} \left| {f_{\eta }^{{''}}(y)} \right|$. Тогда

${{J}^{{({{x}_{I}})}}} \leqslant \frac{1}{h}\mathop {max}\limits_{y \in {{\Delta }^{{({{x}_{I}})}}}} \left| {f_{\eta }^{{''}}(y)} \right|\int\limits_{{{\Delta }^{{({{x}_{I}})}}}} {\frac{{{{{(y - {{x}_{I}})}}^{2}}}}{2}} dy = \frac{{{{h}^{2}}}}{{24}}\mathop {max}\limits_{y \in {{\Delta }^{{({{x}_{I}})}}}} \left| {f_{\eta }^{{''}}(y)} \right|.$

Очевидная индукция по размерности $d$ дает соотношение (4.7). Утверждение 4 доказано.

Теорема 2. Для алгоритма с аппроксимационным базисом Стренга–Фикса (3.1) с образующей функцией (3.4), простейшими коэффициентами (3.6) и ядерной функцией (4.5) справедливы неравенства

(4.8)
$\delta _{{{\text{bias}}}}^{{(\mathbb{C})}}(M) \leqslant H_{{{\text{bias}}}}^{{(\mathbb{C})}}{{M}^{{ - 2/d}}},\quad \delta _{{{\text{bias}}}}^{{({{\mathbb{L}}_{2}})}}(M) \leqslant H_{{{\text{bias}}}}^{{({{\mathbb{L}}_{2}})}}{{M}^{{ - 2/d}}}$
при ${{f}_{\eta }} \in {{\mathbb{C}}^{2}}(X)$ и ${{f}_{\eta }} \in \mathbb{W}_{2}^{2}(X)$ соответственно. Здесь

(4.9)
$H_{{{\text{bias}}}}^{{(\mathbb{C})}} = \frac{{{{{[{{H}^{{(h \to M)}}}]}}^{2}}}}{{24}}\sum\limits_{s = 1}^d {\mathop {max}\limits_{{\mathbf{x}} \in X} } \left| {\frac{{{{\partial }^{2}}}}{{\partial {{{({{x}^{{(s)}}})}}^{2}}}}{{f}_{\eta }}({\mathbf{x}})} \right|\quad и\quad H_{{{\text{bias}}}}^{{({{\mathbb{L}}_{2}})}} = H_{{{\text{bias}}}}^{{(\mathbb{C})}} \times \sqrt {\operatorname{mes} X} .$

Утверждение теоремы 2 следует из соотношений (4.2), (4.4), (4.6), (4.7) и очевидного неравенства $\delta _{{{\text{bias}}}}^{{({{\mathbb{L}}_{2}})}}(M) \leqslant \sqrt {\operatorname{mes} X} \times \delta _{{{\text{bias}}}}^{{(\mathbb{C})}}(M)$.

Сравнивая неравенства (3.8), (3.9) и (4.8), (4.9), можно сделать вывод о том, что упомянутая выше возможность рассмотрения базиса (3.1) с кубической производящей функцией (3.10) для получения порядка ${{M}^{{ - 4/d}}}$ для величин $\delta _{{{\text{det}}}}^{{(\mathbb{C})}}(M)$, $\delta _{{{\text{det}}}}^{{({{\mathbb{L}}_{2}})}}(M)$ для рассматриваемого алгоритма 1 не имеет смысла, так как компоненты смещения имеют порядок ${{M}^{{ - 2/d}}}$ (это аналог основного вывода [24]).

Также можно высказать весьма правдоподобную гипотезу (имеющую некоторые численные подтверждения – у нас имеются соответствующие предварительные результаты тестовых расчетов, но требующую, впрочем, отдельной тщательной проверки) о том, что использование в алгоритме 1 ядерных функций ${{\kappa }^{{({\mathbf{x}})}}}({\mathbf{y}})$, отличных от (4.5) (например, оптимальных функций из работы [7]), не позволит получить верхних границ компонент смещения лучших, чем (4.8).

Из теоремы 2 и двух последних соображений вытекает следующий важный вывод.

Замечание 2. Для решения сформулированной в разделе 1 задачи 1 целесообразным является применение следующего приближения алгоритма 1 с аппроксимационным базисом ${{\Xi }^{{(M)}}}$ вида (3.1) с образующей функцией (3.4), простейшими аппроксимационными коэффициентами вида (3.6) и ядерной функцией ${{\kappa }^{{({\mathbf{x}})}}}({\mathbf{y}})$ вида (4.5):

(4.10)
${{f}_{\eta }}({\mathbf{x}}) \approx {{L}^{{(M,n)}}}{{f}_{\eta }}({\mathbf{x}}) = \sum\limits_{i = 1}^M {\tilde {f}_{\eta }^{{({{{\mathbf{x}}}_{i}})}}} {{\chi }^{{(i,1)}}}({\mathbf{x}});\quad \tilde {f}_{\eta }^{{({{{\mathbf{x}}}_{i}})}} = \frac{1}{{n{{h}^{d}}}}\sum\limits_{j = 1}^n {{{X}^{{({{\Delta }^{{({{{\mathbf{x}}}_{i}})}}})}}}} ({{\eta }_{{\mathbf{j}}}}).$
Здесь ${{X}^{{({{\Delta }^{{({{{\mathbf{x}}}_{i}})}}})}}}({\mathbf{y}})$ – индикатор множества ${{\Delta }^{{({{x}_{i}})}}}$: при ${\mathbf{y}} \in {{\Delta }^{{({{x}_{i}})}}}$ имеем ${{X}^{{({{\Delta }^{{({{{\mathbf{x}}}_{i}})}}})}}}({\mathbf{y}}) = 1$, а при ${\mathbf{y}} \notin {{\Delta }^{{({{x}_{i}})}}}$ выполнено ${{X}^{{({{\Delta }^{{({{{\mathbf{x}}}_{i}})}}})}}}({\mathbf{y}}) = 0$.

По аналогии с теорией рандомизированных функциональных алгоритмов приближения решения интегрального уравнения Фредгольма (см., в частности, [14], [16]–[24]), назовем вариант алгоритма 1, описанный в теореме 2 и в замечании 2 (см. соотношение (4.10)), многомерным аналогом метода полигона частот. Такой термин здесь более чем удачен, так как в одномерном случае $d = 1$ алгоритм 1 из теоремы 2 и замечания 2 дает в точности полигон частот.

В связи с замечанием 2 мы займемся выводом формул для условно-оптимальных параметров именно для многомерного аналога метода полигона частот (4.10). Для простоты везде далее будем полагать, что

(4.11)
${{f}_{\eta }} \in {{\mathbb{C}}^{2}}(X) \subset \mathbb{W}_{2}^{2}(X).$

5. ВЕРХНЯЯ ГРАНИЦА ДЛЯ СТОХАСТИЧЕСКОЙ КОМПОНЕНТЫ ${{\mathbb{L}}_{2}}$-ПОГРЕШНОСТИ

Построим верхние границы стохастических компонент погрешности $\delta _{{{\text{stoch}}}}^{{({{\mathbb{L}}_{2}})}}(M,n)$, $\delta _{{{\text{stoch}}}}^{{(\mathbb{C})}}(M,n)$ из соотношений (2.5), (2.8) для многомерного аналога метода полигона частот (4.10). Докажем сначала следующее вспомогательное утверждение.

Утверждение 5. Для приближения

${{f}_{\eta }}({\mathbf{x}}) \approx {{L}^{{(M,n)}}}{{\tilde {f}}_{\eta }}({\mathbf{x}}) = \sum\limits_{i = 1}^M {\left[ {\frac{1}{n}\sum\limits_{j = 1}^n {{{\kappa }^{{({{{\mathbf{x}}}_{i}})}}}({{\eta }_{{\mathbf{j}}}})} } \right]{\kern 1pt} } {{\chi }^{{(i,1)}}}({\mathbf{x}})$
справедливо соотношение

$\delta _{{{\text{stoch}}}}^{{({{\mathbb{L}}_{2}})}}(M,n) = \int\limits_X {\mathbf{D}} {{L}^{{(M,n)}}}{{\tilde {f}}_{\eta }}({\mathbf{x}})d{\mathbf{x}} \leqslant \frac{{\operatorname{mes} Xma{{x}_{{i = 1, \ldots ,M}}}{\mathbf{D}}{{\kappa }^{{({{{\mathbf{x}}}_{i}})}}}(\eta )}}{n}.$

Доказательство. С учетом известного тождества ${\mathbf{D}}\tau = {\mathbf{D}}(\tau + C)$, $C = {\text{const}}$, справедливого для любой случайной величины $\tau $, имеем

$\begin{gathered} {\mathbf{D}}{{L}^{{(M,n)}}}\mathop {\tilde {f}}\nolimits_\eta ({\mathbf{x}}) = {\mathbf{D}}[{{L}^{{(M,n)}}}{{{\tilde {f}}}_{\eta }}({\mathbf{x}}) - {{L}^{{(M)}}}{{{\bar {f}}}_{\eta }}({\mathbf{x}})] = {\mathbf{D}}\left[ {\sum\limits_{i = 1}^M {\left( {\frac{1}{n}\sum\limits_{j = 1}^n {{{\kappa }^{{({{{\mathbf{x}}}_{i}})}}}} ({{\eta }_{{\mathbf{j}}}})} \right)} } \right.{{\chi }^{{(i,1)}}}({\mathbf{x}}) - \\ - \;\left. {\sum\limits_{i = 1}^M {{\mathbf{E}}{{\kappa }^{{({{{\mathbf{x}}}_{i}})}}}} (\eta ){\kern 1pt} {{\chi }^{{(i,1)}}}({\mathbf{x}})} \right] = {\mathbf{D}}\left[ {\sum\limits_{i = 1}^M {\left( {\frac{1}{n}\sum\limits_{j = 1}^n {\mathop {\tilde {\zeta }}\nolimits_j^{(i)} } } \right)} {\kern 1pt} {{\chi }^{{(i,1)}}}({\mathbf{x}})} \right], \\ \end{gathered} $
где $\mathop {\tilde {\zeta }}\nolimits_j^{(i)} = {{\kappa }^{{({{{\mathbf{x}}}_{i}})}}}({{\eta }_{{\mathbf{j}}}}) - {\mathbf{E}}{{\kappa }^{{({{{\mathbf{x}}}_{i}})}}}({{\eta }_{{\mathbf{j}}}})$ (здесь мы трактуем ${{\eta }_{1}},\; \ldots ,\;{{\eta }_{{\mathbf{n}}}}$ как независимые, одинаково распределенные – как $\eta $ – случайные величины).

Зафиксируем ${\mathbf{x}} = {{{\mathbf{z}}}_{0}} \in X$. Учитывая, что ${\mathbf{E}}\mathop {\tilde {\zeta }}\nolimits_j^{(i)} = 0$ для всех $i = 1,\;2,\; \ldots ,\;M$ и $j = 1,\;2,\; \ldots ,\;n$, получаем равенство

${\mathbf{D}}{{L}^{{(M,n)}}}{{\tilde {f}}_{\eta }}({{{\mathbf{z}}}_{0}}) = \sum\limits_{i = 1}^M {\mathbf{D}} \left[ {\frac{{{{\chi }^{{(i,1)}}}({{{\mathbf{z}}}_{0}})}}{n}\sum\limits_{j = 1}^n {\tilde {\zeta }_{j}^{{(i)}}} } \right] + \frac{1}{n}\int\limits_{{{i}_{1}},{{i}_{2}} = 1, \ldots ,M;{{i}_{1}} \ne {{i}_{2}}} {{{\chi }^{{({{i}_{1}},1)}}}} ({{{\mathbf{z}}}_{0}}){{\chi }^{{({{i}_{2}},1)}}}({{{\mathbf{z}}}_{0}}){\mathbf{E}}[{{\tilde {\zeta }}^{{({{i}_{1}})}}}{{\tilde {\zeta }}^{{({{i}_{2}})}}}],$
где ${{\tilde {\zeta }}^{{(i)}}}$ – случайная величина, распределенная как независимые для разных $j$, одинаково распределенные величины $\tilde {\zeta }_{j}^{{(i)}}$.

Далее, применяя вероятностный аналог неравенства Коши–Буняковского

${\mathbf{E}}\left| {{{{\tilde {\zeta }}}^{{({{i}_{1}})}}}{{{\tilde {\zeta }}}^{{({{i}_{2}})}}}} \right| \leqslant \sqrt {{\mathbf{E}}{{{[{{{\tilde {\zeta }}}^{{({{i}_{1}})}}}]}}^{2}} \times {\mathbf{E}}{{{[{{{\tilde {\zeta }}}^{{({{i}_{2}})}}}]}}^{2}}} = \sqrt {{\mathbf{D}}{{{\tilde {\zeta }}}^{{({{i}_{1}})}}} \times {\mathbf{D}}{{{\tilde {\zeta }}}^{{({{i}_{2}})}}}} $
и соотношения ${\mathbf{D}}{{\tilde {\zeta }}^{{(i)}}} = {\mathbf{D}}{{\kappa }^{{({{{\mathbf{x}}}_{i}})}}}(\eta )$ для $i = 1,\;2,\; \ldots ,\;M$ получаем оценку сверху
$\begin{gathered} {\mathbf{D}}{{L}^{{(M,n)}}}{{{\tilde {f}}}_{\eta }}({{{\mathbf{z}}}_{0}}) \leqslant \frac{{ma{{x}_{{i = 1, \ldots ,M}}}{\mathbf{D}}{{\kappa }^{{({{{\mathbf{x}}}_{i}})}}}(\eta )}}{n}\left[ {\sum\limits_{i = 1}^M {{{{({{\chi }^{{(i,1)}}})}}^{2}}} ({{{\mathbf{z}}}_{0}})} \right. + \\ + \;\left. {\sum\limits_{{{i}_{1}},{{i}_{2}} = 1, \ldots ,M;{{i}_{1}} \ne {{i}_{2}}} {{{\chi }^{{({{i}_{1}},1)}}}({{{\mathbf{z}}}_{0}}){{\chi }^{{({{i}_{2}},1)}}}({{{\mathbf{z}}}_{0}})} } \right] = \frac{{ma{{x}_{{i = 1, \ldots ,M}}}{\mathbf{D}}{{\kappa }^{{({{{\mathbf{x}}}_{i}})}}}\left( \eta \right)}}{n}{{\left[ {\sum\limits_{i = 1}^M {{{\chi }^{{(i,1)}}}({{{\mathbf{z}}}_{0}})} } \right]}^{2}}. \\ \end{gathered} $
Наконец, в силу утверждения 3 и произвольности ${{{\mathbf{z}}}_{0}} \in X$, имеем
$\mathop {sup}\limits_{{\mathbf{x}} \in X} {\mathbf{D}}{{L}^{{(M,n)}}}{{\tilde {f}}_{\eta }}({\mathbf{x}}) \leqslant \frac{{ma{{x}_{{i = 1, \ldots ,M}}}{\mathbf{D}}{{\kappa }^{{({{{\mathbf{x}}}_{i}})}}}(\eta )}}{n}\quad {\text{и}}\quad \delta _{{{\text{stoch}}}}^{{({{\mathbb{L}}_{2}})}}(M,n) \leqslant \frac{{\operatorname{mes} X{{{\max }}_{{i = 1, \ldots ,M}}}{\mathbf{D}}{{\kappa }^{{({{{\mathbf{x}}}_{i}})}}}(\eta )}}{n}.$
Утверждение 5 доказано.

Теорема 3. Для многомерного аналога полигона частот (4.10) (т.е. для алгоритма 1 с аппроксимационным базисом ${{\Xi }^{{(M)}}}$ вида (3.1) с образующей функцией (3.4), простейшими аппроксимационными коэффициентами вида (3.6) и ядерной функцией ${{\kappa }^{{({\mathbf{x}})}}}({\mathbf{y}})$ вида (4.5)) при выполнении условия (4.11) справедливы неравенства

(5.1)
$\mathop {max}\limits_{i = 1, \ldots ,M} {\mathbf{D}}{{\kappa }^{{({{{\mathbf{x}}}_{i}})}}}\left( \eta \right) \leqslant \frac{{{{f}_{{{\text{max}}}}} \times M}}{{{{{[{{H}^{{(h \to M)}}}]}}^{d}}}}\quad и\quad \delta _{{{\text{stoch}}}}^{{({{\mathbb{L}}_{2}})}}(M,n) \leqslant \frac{{H_{{{\text{stoch}}}}^{{({{\mathbb{L}}_{2}})}}M}}{n},$
где

(5.2)
$H_{{{\text{stoch}}}}^{{({{\mathbb{L}}_{2}})}} = \frac{{{{f}_{{{\text{max}}}}} \times \operatorname{mes} X}}{{{{{[{{H}^{{(h \to M)}}}]}}^{d}}}}\quad и\quad {{f}_{{{\text{max}}}}} = \mathop {max}\limits_{{\mathbf{x}} \in X} {{f}_{\eta }}({\mathbf{x}}).$

Доказательство. Для ядерной функции (4.5) имеем

${\mathbf{D}}{{\kappa }^{{({{{\mathbf{x}}}_{i}})}}}(\eta ) = \frac{{{{I}_{i}} - I_{i}^{2}}}{{{{h}^{{2d}}}}},\quad {\text{где}}\quad {{I}_{i}} = \int\limits_{{{\Delta }^{{({{{\mathbf{x}}}_{i}})}}}} {{{f}_{\eta }}({\mathbf{y}})d{\mathbf{y}}} .$

Учитывая условие (4.11) и теорему о среднем для определенного интеграла от непрерывной функции, получаем, что существует ${{{\mathbf{z}}}_{i}} \in {{\Delta }^{{({{{\mathbf{x}}}_{i}})}}}$ такое, что

(5.3)
${{I}_{i}} = {{f}_{\eta }}({{{\mathbf{z}}}_{i}}) \times {{h}^{d}}\quad {\text{и}}\quad {\mathbf{D}}{{\kappa }^{{({{{\mathbf{x}}}_{i}})}}}(\eta ) = \frac{{{{f}_{\eta }}({{{\mathbf{z}}}_{i}})}}{{{{h}^{d}}}} - f_{\eta }^{2}({{{\mathbf{z}}}_{i}}),$
и

(5.4)
${{f}_{\eta }}({{{\mathbf{z}}}_{i}}) = {\mathbf{E}}{{\kappa }^{{({{{\mathbf{x}}}_{i}})}}}(\eta ).$

В силу ограниченности функции ${{f}_{\eta }} \in {{\mathbb{C}}^{2}}(X)$ на компакте $X$, из утверждения 5 и соотношений (3.2), (5.3) получаем соотношения (5.1), (5.2). Теорема 3 доказана.

6. ВЕРХНЯЯ ГРАНИЦА ДЛЯ СТОХАСТИЧЕСКОЙ КОМПОНЕНТЫ $\mathbb{C}$-ПОГРЕШНОСТИ

Для $\mathbb{C}$-подхода, используя соотношение (4.3), получаем

$\begin{gathered} \delta _{{{\text{stoch}}}}^{{(\mathbb{C})}}(M,n) \leqslant \mathop {max}\limits_{i = 1,...,M} \left| {{{Z}_{n}}({{{\mathbf{x}}}_{i}}) - {\mathbf{E}}{{\kappa }^{{({{{\mathbf{x}}}_{i}})}}}(\eta )} \right| = \mathop {max}\limits_{i = 1,...,M} \left| {\sum\limits_{j = 1}^n {\frac{{{{\kappa }^{{({{{\mathbf{x}}}_{i}})}}}({{\eta }_{{\mathbf{j}}}}) - n{\mathbf{E}}{{\kappa }^{{({{{\mathbf{x}}}_{i}})}}}(\eta )}}{n}} } \right| = \\ = \mathop {max}\limits_{i = 1,...,M} \left| {\sqrt {\frac{{{\mathbf{D}}{{\kappa }^{{({{{\mathbf{x}}}_{i}})}}}(\eta )}}{n}} \times \sum\limits_{j = 1}^n {\frac{{{{\kappa }^{{({{{\mathbf{x}}}_{i}})}}}({{\eta }_{{\mathbf{j}}}}) - n{\mathbf{E}}{{\kappa }^{{({{{\mathbf{x}}}_{i}})}}}(\eta )}}{{\sqrt {{\mathbf{D}}{{\kappa }^{{({{{\mathbf{x}}}_{i}})}}}(\eta ) \times n} }}} } \right|. \\ \end{gathered} $

С учетом соотношений (5.1) имеем

$\delta _{{{\text{stoch}}}}^{{(\mathbb{C})}}(M,n) \leqslant \sqrt {\frac{{{{f}_{{{\text{max}}}}} \times M}}{{{{{[{{H}^{{(h \to M)}}}]}}^{d}} \times n}}} \times \mathop {max}\limits_{i = 1,...,M} \left| {\sum\limits_{j = 1}^n {\frac{{{{\kappa }^{{({{{\mathbf{x}}}_{i}})}}}({{\eta }_{{\mathbf{j}}}}) - n{\mathbf{E}}{{\kappa }^{{({{{\mathbf{x}}}_{i}})}}}(\eta )}}{{\sqrt {{\mathbf{D}}{{\kappa }^{{({{{\mathbf{x}}}_{i}})}}}(\eta ) \times n} }}} } \right|.$

В силу центральной предельной теоремы для одинаково распределенных случайных величин, суммы

${{\omega }^{{(i,M)}}}(n) = \sum\limits_{j = 1}^n {\frac{{{{\kappa }^{{({{{\mathbf{x}}}_{i}})}}}({{\eta }_{{\mathbf{j}}}}) - n{\mathbf{E}}{{\kappa }^{{({{{\mathbf{x}}}_{i}})}}}(\eta )}}{{\sqrt {{\mathbf{D}}{{\kappa }^{{({{{\mathbf{x}}}_{i}})}}}(\eta ) \times n} }}} $
сходятся (по распределению) к стандартным гауссовским величинам ${{\gamma }^{{(i,M)}}}$ при $n \to \infty $. Следовательно, при фиксированном $M$ для любого ${{\varepsilon }_{1}} > 0$ найдется такое натуральное ${{N}_{1}}({{\varepsilon }_{1}},M)$, что при $n > {{N}_{1}}({{\varepsilon }_{1}},M)$ выполнено
(6.1)
${\mathbf{P}}\left\{ {\delta _{{{\text{stoch}}}}^{{(\mathbb{C})}}(M,n) \leqslant \frac{{H_{{{\text{stoch}},1}}^{{(\mathbb{C})}}({{\varepsilon }_{1}})\sqrt M }}{{\sqrt n }} \times \mathop {max}\limits_{i = 1,...,M} \left| {{{\gamma }^{{(i,M)}}}} \right|} \right\} > 1 - {{\varepsilon }_{1}},$
где
(6.2)
$H_{{{\text{stoch}},1}}^{{(\mathbb{C})}}({{\varepsilon }_{1}}) = {{B}_{1}}({{\varepsilon }_{1}})\sqrt {\frac{{{{f}_{{{\text{max}}}}}}}{{{{{[{{H}^{{(h \to M)}}}]}}^{d}}}}} .$
Здесь ${{B}_{1}}({{\varepsilon }_{1}})$ – небольшая положительная константа, такая, что $\int_{ - {{B}_{1}}({{\varepsilon }_{1}})}^{{{B}_{1}}({{\varepsilon }_{1}})} {\tfrac{{{{e}^{{ - {{{v}}^{2}}/2}}}d{v}}}{{\sqrt {2\pi } }}} \approx 1 - {{\varepsilon }_{1}}$.

Подробному изучению распределений и асимптотических (при $M \to \infty $) свойств величин типа ${{Y}^{{(M)}}} = {{\max }_{{i = 1,...,M}}}{{\gamma }^{{(i,M)}}}$ посвящена монография (см. [27]). Некоторая особенность рассматриваемого здесь случая состоит в том, что величины $\{ {{\gamma }^{{(i,M)}}};i = 1,\;2,\; \ldots ,\;M\} $ являются зависимыми. Однако эта зависимость “ослабевает” с ростом $M$. Конкретнее, для ковариаций

(6.3)
$\operatorname{cov} [{{\gamma }^{{({{i}_{1}},M)}}},{{\gamma }^{{({{i}_{2}},M)}}}] = {\mathbf{E}}{{\gamma }^{{({{i}_{1}},M)}}}{{\gamma }^{{({{i}_{2}},M)}}},\quad {{i}_{1}} \ne {{i}_{2}},\quad {{i}_{1}},{{i}_{2}} = 1,\;2,\; \ldots ,\;M,$
справедливы соотношения
$\begin{gathered} \left| {\operatorname{cov} \text{[}{{\gamma }^{{({{i}_{1}},M)}}},{{\gamma }^{{({{i}_{2}},M)}}}]} \right| = \left| {\operatorname{cov} [{{\omega }^{{({{i}_{1}},M)}}}(n),{{\omega }^{{({{i}_{2}},M)}}}(n)]} \right| = \left| {\frac{{{\mathbf{E}}{{\kappa }^{{({{{\mathbf{x}}}_{{{{i}_{1}}}}})}}}(\eta ){{\kappa }^{{({{{\mathbf{x}}}_{{{{i}_{2}}}}})}}}(\eta ) - {\mathbf{E}}{{\kappa }^{{({{{\mathbf{x}}}_{{{{i}_{1}}}}})}}}(\eta ){\mathbf{E}}{{\kappa }^{{({{{\mathbf{x}}}_{{{{i}_{2}}}}})}}}(\eta )}}{{\sqrt {{\mathbf{D}}{{\kappa }^{{({{{\mathbf{x}}}_{{{{i}_{1}}}}})}}}(\eta ){\mathbf{D}}{{\kappa }^{{({{{\mathbf{x}}}_{{{{i}_{2}}}}})}}}(\eta )} }}} \right| = \\ = \frac{{{{{[{{H}^{{(h \to M)}}}]}}^{d}}\sqrt {{{f}_{\eta }}({{{\mathbf{z}}}_{{{{i}_{1}}}}}){{f}_{\eta }}({{{\mathbf{z}}}_{{{{i}_{2}}}}})} }}{{\sqrt {{{M}^{2}} - M{{{[{{H}^{{(h \to M)}}}]}}^{d}}[{{f}_{\eta }}({{{\mathbf{z}}}_{{{{i}_{1}}}}}) + {{f}_{\eta }}({{{\mathbf{z}}}_{{{{i}_{2}}}}})] + {{{[{{H}^{{(h \to M)}}}]}}^{{2d}}}{{f}_{\eta }}({{{\mathbf{z}}}_{{{{i}_{1}}}}}){{f}_{\eta }}({{{\mathbf{z}}}_{{{{i}_{2}}}}})} }}. \\ \end{gathered} $
Здесь использованы соотношения (3.2), (5.3), (5.4) и то, что для ядерной функции (4.5) выполнено ${{\kappa }^{{({{{\mathbf{x}}}_{{{{i}_{1}}}}})}}}(\eta ) \times {{\kappa }^{{({{{\mathbf{x}}}_{{{{i}_{2}}}}})}}}(\eta ) \equiv 0$ при ${{i}_{1}} \ne {{i}_{2}}$.

Из последнего соотношения следует, что ковариации (6.3) убывают по модулю с ростом $M$ со скоростью порядка $1{\text{/}}M$; при этом найдутся натуральное число ${{\hat {M}}_{1}}$ (например, ${{\hat {M}}_{1}} = 2{{f}_{{{\text{max}}}}}{{[{{H}^{{(h \to M)}}}]}^{d}}$) и “универсальная” константа ${{B}_{2}} > 0$ (например, ${{B}_{2}} = \sqrt 2 {{f}_{{{\text{max}}}}}{{[{{H}^{{(h \to M)}}}]}^{d}}$) такие, что

(6.4)
$\left| {\operatorname{cov} [{{\gamma }^{{({{i}_{1}},M)}}},{{\gamma }^{{({{i}_{2}},M)}}}]} \right| \leqslant \frac{{{{B}_{2}}}}{M}\quad {\text{при}}\quad M > {{\hat {M}}_{1}}.$

Используя технологию изучения порядковых статистик из [27] по аналогии с [22], можно получить следующий результат.

Утверждение 6 (см. [22], [27]). Если для набора стандартных нормальных случайных величин ${{\gamma }^{{(1,M)}}},\; \ldots ,\;{{\gamma }^{{(M,M)}}}$ выполнены соотношения (6.4), то асимптотическое при $M \to \infty $ распределение максимума ${{Y}^{{(M)}}} = ma{{x}_{{i = 1,...,M}}}{{\gamma }^{{(i,M)}}}$ таково, что

${\mathbf{P}}\left\{ {{{a}_{M}}({{Y}^{{(M)}}} - {{b}_{M}}) < y} \right\} \to \exp ( - {{e}^{{ - y}}}) = G(y),$
где

(6.5)
${{a}_{M}} = \sqrt {2lnM} ,\quad {{b}_{M}} = \sqrt {2lnM} - \frac{{lnlnM + ln4\pi }}{{2\sqrt {2lnM} }}.$

В силу симметрии стандартного нормального распределения и с учетом соотношений

$\mathop {max}\limits_{i = 1,..,M} \left| {{{\gamma }^{{(i,M)}}}} \right| = max\left( {\mathop {max}\limits_{i = 1,...,M} {{\gamma }^{{(i,M)}}},\; - {\kern 1pt} \mathop {min}\limits_{i = 1,...,M} {{\gamma }^{{(i,M)}}}} \right),$
$min\{ {{\gamma }^{{(1,M)}}},\; \ldots ,\;{{\gamma }^{{(M,M)}}}\} = - max\{ - {{\gamma }^{{(1,M)}}},\; \ldots ,\; - {\kern 1pt} {{\gamma }^{{(M,M)}}}\} $
получаем, что
${\mathbf{P}}\left\{ {{{a}_{M}}\left( {\mathop {max}\limits_{i = 1,...,M} \left| {{{\gamma }^{{(i,M)}}}} \right| - {{b}_{M}}} \right) < y} \right\} \to \exp ( - 2{{e}^{{ - y}}}) = {{G}^{2}}(y),$
где коэффициенты ${{a}_{M}}$, ${{b}_{M}}$ имеют вид (6.5).

Из последнего соотношения следует, что для любого ${{\varepsilon }_{2}} > 0$ существуют константа ${{B}_{3}}({{\varepsilon }_{2}}) > 0$ и натуральное число ${{\hat {M}}_{2}}$ такие, что

(6.6)
${\mathbf{P}}\left\{ {\mathop {max}\limits_{i = 1,...,M} \left| {{{\gamma }^{{(i,M)}}}} \right| \leqslant \frac{{{{B}_{3}}({{\varepsilon }_{2}})}}{{{{a}_{M}}}} + {{b}_{M}}} \right\} > 1 - {{\varepsilon }_{2}}\quad {\text{при}}\quad M > {{\hat {M}}_{2}}.$

С учетом соотношений (6.1), (6.2), (6.6) получаем следующий результат.

Теорема 4. Для многомерного аналога полигона частот (4.10) (т.е. для алгоритма с аппроксимационным базисом ${{\Xi }^{{(M)}}}$ вида (3.1) с образующей функцией (3.4), простейшими аппроксимационными коэффициентами вида (3.6) и ядерной функцией ${{\kappa }^{{({\mathbf{x}})}}}({\mathbf{y}})$ вида (4.5)) для любого $\varepsilon > 0$ существуют положительные действительные константы $H_{{{\text{stoch}},1}}^{{(\mathbb{C})}}(\varepsilon )$, $H_{{{\text{stoch}},2}}^{{(\mathbb{C})}}(\varepsilon )$ и натуральное число $\hat {M}$ такие, что для любого $M > \hat {M}$ существует натуральное число $\hat {N}(\varepsilon ,M)$ такое, что для всех $n > \hat {N}(\varepsilon ,M)$ выполнено

(6.7)
${\mathbf{P}}\left\{ {\delta _{{{\text{stoch}}}}^{{(\mathbb{C})}}(M,n) \leqslant \frac{{H_{{{\text{stoch}},1}}^{{(\mathbb{C})}}(\varepsilon )\sqrt M }}{{\sqrt n }} \times \left[ {\sqrt {2lnM} + \frac{{H_{{{\text{stoch}},2}}^{{(\mathbb{C})}}(\varepsilon ) - \tfrac{{\ln \ln M}}{2}}}{{\sqrt {2lnM} }}} \right]} \right\} > 1 - \varepsilon .$

7. УСЛОВНО-ОПТИМАЛЬНЫЕ ПАРАМЕТРЫ ДЛЯ ${{\mathbb{L}}_{2}}$- И $\mathbb{C}$-ПОДХОДОВ

Полученные в теоремах 1–4 верхние границы (3.8), (4.8), (5.1), (6.7) для компонент погрешности ${{\delta }^{{(\mathbb{B})}}}$ позволяют вывести выражения для условно-оптимальных параметров $M_{{{\text{opt}}}}^{{(\mathbb{B})}}(L)$ и $n_{{{\text{opt}}}}^{{(\mathbb{B})}}(L)$ для $\mathbb{B}(X) = \mathbb{C}(X) \vee {{\mathbb{L}}_{2}}(X)$ по представленному в разд. 2 методу 1.

Важной отличительной особенностью алгоритма 1 является то, что затраты $S$ этого алгоритма пропорциональны величине $n \times t$, где $t$ – время определения того, в какой из кубов ${{\Delta }^{{({{{\mathbf{x}}}_{i}})}}}$ попадает очередное выборочное значение ${{\eta }_{{\mathbf{j}}}}$ случайной величины $\eta $, $i = 1,\;2,\; \ldots ,\;M$, $j = 1,\;2,\; \ldots ,\;n$.

Несложно добиться, чтобы время $t$ не зависело от числа $M$ кубов ${{\Delta }^{{({{{\mathbf{x}}}_{i}})}}}$. Таким образом, затраты оптимизированных версий алгоритма 1 явно не зависят от $M$ (наши тестовые и прикладные численные эксперименты подтверждают это утверждение).

В оптимизационных процедурах метода 1 зависимость затрат $S$ от параметра $M$ возникает из-за наличия уравнений вида (2.2), определяющих зависимость параметра $n$ (а значит, и затрат $S$) от $M$.

Рассмотрим такую процедуру для ${{\mathbb{L}}_{2}}$-подхода. Здесь требуется решать следующую оптимизационную задачу: найти значения $M_{{{\text{opt}}}}^{{({{\mathbb{L}}_{2}})}}(L)$ и $n_{{{\text{opt}}}}^{{({{\mathbb{L}}_{2}})}}(L)$, для которых достигается минимум

(7.1)
$\mathop {min}\limits_{M,n} S(M,n) = \mathop {\min }\limits_{M,n} [t \times n(M,L) + {{S}_{0}}]$
(здесь ${{S}_{0}}$ – время вычисления ${{2}^{d}}$ ненулевых слагаемых $\tilde {f}_{\eta }^{{({{{\mathbf{x}}}_{i}})}}{{\chi }^{{(i,1)}}}({\mathbf{x}})$ в сумме (4.10) для заданной точки ${\mathbf{x}} \in X$) при условии
(7.2)
$\frac{{A_{1}^{{({{\mathbb{L}}_{2}})}}}}{{{{M}^{{4/d}}}}} + \frac{{A_{2}^{{({{\mathbb{L}}_{2}})}}M}}{n}{{L}^{2}},$
где $A_{1}^{{({{\mathbb{L}}_{2}})}} = 2{{(H_{{{\text{stoch}}}}^{{({{\mathbb{L}}_{2}})}})}^{2}} + 2{{(H_{{{\text{bias}}}}^{{({{\mathbb{L}}_{2}})}})}^{2}}$ и $A_{2}^{{({{\mathbb{L}}_{2}})}} = H_{{{\text{stoch}}}}^{{({{\mathbb{L}}_{2}})}}$ (см. соотношения (2.8), (3.8), (3.9), (4.8), (4.9), (5.1), (5.2)).

Из равенств (7.1) и (7.2) имеем

$n = \frac{{A_{2}^{{({{\mathbb{L}}_{2}})}}M}}{{{{L}^{2}} - A_{1}^{{({{\mathbb{L}}_{2}})}}{\text{/}}{{M}^{{4/d}}}}}\quad {\text{и}}\mathop {\quad \tilde {S}}\nolimits^{({{\mathbb{L}}_{2}},L)} (M) = \frac{{tA_{2}^{{({{\mathbb{L}}_{2}})}}M}}{{{{L}^{2}} - A_{1}^{{({{\mathbb{L}}_{2}})}}{\text{/}}{{M}^{{4/d}}}}} + {{S}_{0}}.$

Найдем теперь минимум функции $\mathop {\tilde {S}}\nolimits^{({{\mathbb{L}}_{2}},L)} (M)$ по $M$. Продифференцируем эту функцию:

$\frac{{\partial {{{\tilde {S}}}^{{({{\mathbb{L}}_{2}},L)}}}(M)}}{{\partial M}}tA_{2}^{{({{\mathbb{L}}_{2}})}}{{\left[ {{{L}^{2}} - \frac{{A_{1}^{{({{\mathbb{L}}_{2}})}}}}{{{{M}^{{4/d}}}}} - \frac{{4A_{1}^{{({{\mathbb{L}}_{2}})}}}}{{d \times {{M}^{{4/d}}}}}} \right]} \mathord{\left/ {\vphantom {{\left[ {{{L}^{2}} - \frac{{A_{1}^{{({{\mathbb{L}}_{2}})}}}}{{{{M}^{{4/d}}}}} - \frac{{4A_{1}^{{({{\mathbb{L}}_{2}})}}}}{{d \times {{M}^{{4/d}}}}}} \right]} {{{{\left[ {{{L}^{2}} - \frac{{A_{1}^{{({{\mathbb{L}}_{2}})}}}}{{{{M}^{{4/d}}}}}} \right]}}^{2}}}}} \right. \kern-0em} {{{{\left[ {{{L}^{2}} - \frac{{A_{1}^{{({{\mathbb{L}}_{2}})}}}}{{{{M}^{{4/d}}}}}} \right]}}^{2}}}},$
и найдем критическое значение (точку минимума), а затем, и условно-оптимальные параметры для ${{\mathbb{L}}_{2}}$-подхода:

(7.3)
$M_{{{\text{opt}}}}^{{({{\mathbb{L}}_{2}})}}(L) = {{[A_{1}^{{({{\mathbb{L}}_{2}})}}]}^{{d/4}}}{{\left( {\frac{{d + 4}}{d}} \right)}^{{d/4}}}{{L}^{{ - d/2}}},$
(7.4)
$n_{{{\text{opt}}}}^{{({{\mathbb{L}}_{2}})}}(L)\frac{{A_{2}^{{({{\mathbb{L}}_{2}})}}{{{[A_{1}^{{({{\mathbb{L}}_{2}})}}]}}^{{d/4}}}{{{(d + 4)}}^{{d/4 + 1}}}}}{{4{{d}^{{d/4}}}}}{{L}^{{ - 2 - d/2}}}.$

Для $\mathbb{C}$-подхода решается задача нахождения минимума (7.1) при условии

(7.5)
$\frac{{A_{1}^{{(\mathbb{C})}}}}{{{{M}^{{2/d}}}}} + \frac{{A_{2}^{{(\mathbb{C})}}\sqrt M }}{{\sqrt n }}\left[ {\sqrt {2lnM} + \frac{{A_{3}^{{(\mathbb{C})}} - \tfrac{{\ln \ln M}}{2}}}{{\sqrt {2lnM} }}} \right] = L,$
где $A_{1}^{{(\mathbb{C})}} = H_{{{\text{det}}}}^{{(\mathbb{C})}} + H_{{{\text{bias}}}}^{{(\mathbb{C})}}$, $A_{2}^{{(\mathbb{C})}} = H_{{{\text{stoch}},1}}^{{(\mathbb{C})}}(\varepsilon )$, $A_{3}^{{(\mathbb{C})}} = H_{{{\text{stoch}},2}}^{{(\mathbb{C})}}(\varepsilon )$ (см. соотношения (2.5), (3.8), (3.9), (4.8), (4.9), (6.2), (6.7)).

Выражение в левой части уравнения (7.5) является относительно сложным, и мы применим следующий прием из [22], позволяющий заменить это выражение на более простое. Используя промежуточный результат из доказательства теоремы 1.5.3 из [27], вместо (7.5) можно рассмотреть соотношение

(7.6)
$\frac{{A_{1}^{{(\mathbb{C})}}}}{{{{M}^{{2/d}}}}} + \frac{{A_{2}^{{(\mathbb{C})}}\sqrt M }}{{\sqrt n }}\sqrt {2lnM - \ln \ln M + 2A_{3}^{{(\mathbb{C})}}} = L.$

Несложно показать, что для любых фиксированных $\bar {M} \in \mathbb{N}$ и $a > 0$ найдется $b > 0$ такое, что при $M > \bar {M}$ выполнено

(7.7)
$\sqrt {2lnM - \ln \ln M + 2A_{3}^{{(\mathbb{C})}}} \leqslant b{{M}^{a}}.$

С учетом соотношений (7.6), (7.7) заменим соотношение (7.5) на следующее:

(7.8)
$\frac{{A_{1}^{{(\mathbb{C})}}}}{{{{M}^{{2/d}}}}} + \frac{{A_{2}^{{(\mathbb{C})}}\sqrt M }}{{\sqrt n }}b{{M}^{a}} = L.$

Из формул (7.1), (7.8) имеем

(7.9)
$n = \frac{{{{{[A_{2}^{{(\mathbb{C})}}]}}^{2}}{{b}^{2}}{{M}^{{2a + 1}}}}}{{{{{[L - A_{1}^{{(\mathbb{C})}}{\text{/}}{{M}^{{2/d}}}]}}^{2}}}}\quad {\text{и}}\quad {{\tilde {S}}^{{(\mathbb{C},L)}}}(M) = \frac{{t{{{[A_{2}^{{(\mathbb{C})}}]}}^{2}}{{b}^{2}}{{M}^{{2a + 1}}}}}{{{{{[L - A_{1}^{{(\mathbb{C})}}{\text{/}}{{M}^{{2/d}}}]}}^{2}}}} + {{S}_{0}}.$

Дифференцируем ${{\tilde {S}}^{{(\mathbb{C},L)}}}(M)$ по $M$:

$\frac{{\partial {{{\tilde {S}}}^{{(\mathbb{C},L)}}}(M)}}{{\partial M}}\frac{{t{{{[A_{2}^{{(\mathbb{C})}}]}}^{2}}{{b}^{2}}(2a + 1){{M}^{{2a + 1}}}}}{{{{{[L - A_{1}^{{(\mathbb{C})}}{\text{/}}{{M}^{{2/d}}}]}}^{3}}}} \times \left( {L - \frac{{A_{1}^{{(\mathbb{C})}}[(2a + 1)d + 4]}}{{{{M}^{{2/d}}}(2a + 1)d}}} \right),$
и найдем точку минимума – она же условно-оптимальное значение параметра $M$ для $\mathbb{C}$-подхода

(7.10)
$M_{{{\text{opt}}}}^{{(\mathbb{C})}}(L) = \mathop {\left( {\frac{{A_{1}^{{(\mathbb{C})}}[(2a + 1)d + 4]}}{{(2a + 1)d}}} \right)}\nolimits^{d/2} {{L}^{{ - d/2}}}.$

Несколько слов о том, как выбирать параметр $b$. С одной стороны, его следует брать по возможности малым, так как множитель ${{b}^{2}}$ входит в выражение для трудоемкости (7.9). С другой стороны, для значения $M_{{{\text{opt}}}}^{{(\mathbb{C})}}(L)$ должно выполняться неравенство (7.7). Поэтому выберем $b$ из условия равенства в соотношении (7.7) при $M = M_{{{\text{opt}}}}^{{(\mathbb{C})}}(L)$:

(7.11)
${{b}^{2}} = \frac{{2lnM_{{{\text{opt}}}}^{{(\mathbb{C})}}(L) - \ln \ln M_{{{\text{opt}}}}^{{(\mathbb{C})}}(L) + 2A_{3}^{{(\mathbb{C})}}}}{{{{{[M_{{{\text{opt}}}}^{{(\mathbb{C})}}(L)]}}^{{2a}}}}}.$

Из соотношений (7.9)–(7.11) получаем условно-оптимальное значение параметра $n$ для $\mathbb{C}$-подхода

(7.12)
$n_{{{\text{opt}}}}^{{(\mathbb{C})}}(L) = \frac{{{{{[A_{1}^{{(\mathbb{C})}}]}}^{{d/2}}}{{{[A_{2}^{{(\mathbb{C})}}]}}^{2}}{{{[(2a + 1)d + 4]}}^{{2 + d/2}}}}}{{16{{{[(2a + 1)d]}}^{{d/2}}}}}[2lnM_{{{\text{opt}}}}^{{(\mathbb{C})}}(L) - \ln \ln M_{{{\text{opt}}}}^{{(\mathbb{C})}}(L) + 2A_{3}^{{(\mathbb{C})}}]{{L}^{{ - 2 - d/2}}}.$

Наконец, для важного предельного случая $a = 0$ имеем

$M_{{{\text{opt}}}}^{{(\mathbb{C})}}(L) = {{\left[ {\frac{{A_{1}^{{(\mathbb{C})}}(d + 4)}}{d}} \right]}^{{d/2}}}{{L}^{{ - d/2}}},$
$n_{{{\text{opt}}}}^{{(\mathbb{C})}}(L) = \frac{{{{{[A_{1}^{{(\mathbb{C})}}]}}^{{d/2}}}{{{[A_{2}^{{(\mathbb{C})}}]}}^{2}}{{{(d + 4)}}^{{2 + d/2}}}}}{{16{{d}^{{d/2}}}}}[2lnM_{{{\text{opt}}}}^{{(\mathbb{C})}}(L) - \ln \ln M_{{{\text{opt}}}}^{{(\mathbb{C})}}(L) + 2A_{3}^{{(\mathbb{C})}}]{{L}^{{ - 2 - d/2}}}.$

Замечание 3. Формулы (7.10), (7.12) заметно сложнее формул (7.3), (7.4). Поэтому на практике целесообразно использовать ${{\mathbb{L}}_{2}}$-подход к условной оптимизации алгоритма 1. Данное замечание вполне соответствует рекомендациям из [7]–[9].

Значения в правых частях соотношений (7.3), (7.4), (7.10), (7.12) могут быть нецелыми, поэтому на практике в качестве параметров $M_{{{\text{opt}}}}^{{({{\mathbb{L}}_{2}})}}(L)$, $n_{{{\text{opt}}}}^{{({{\mathbb{L}}_{2}})}}(L)$, $M_{{{\text{opt}}}}^{{(\mathbb{C})}}(L)$, $n_{{{\text{opt}}}}^{{(\mathbb{C})}}(L)$ следует брать ближайшие к этим значениям целые числа.

Отметим также наличие содержательной проблемы выбора или приближенного подсчета констант (3.9), (4.9), (5.2), (6.2), входящих в выражения (7.1)–(7.12).

При обсуждении результатов данной работы было отмечено, что использование в одномерном случае максимума $\mathop {{\text{max}}}\nolimits_{x \in X} \left| {\tfrac{{{{\partial }^{2}}}}{{\partial {{x}^{2}}}}{{f}_{\eta }}(x)} \right|$ в оценках сверху для детерминированной компоненты $\delta _{{{\text{det}}}}^{{({{\mathbb{L}}_{2}})}}(M)$ и компоненты смещения $\delta _{{{\text{bias}}}}^{{({{\mathbb{L}}_{2}})}}(M)$ (см. соотношения (3.9) и (4.9)) кажется достаточно “жестким” по сравнению с оценками вида

${{[\tilde {\delta }_{{{\text{bias}}}}^{{({{\mathbb{L}}_{2}})}}(M)]}^{2}} \leqslant \tilde {H}_{{{\text{bias}}}}^{{({{\mathbb{L}}_{2}})}}{{M}^{{ - 4}}},\quad \mathop {\tilde {H}}\nolimits_{{\text{bias}}}^{({{\mathbb{L}}_{2}})} = \frac{{{{{[{{H}^{{(h \to M)}}}]}}^{4}}}}{{576}}\int\limits_X {{{{\left[ {\frac{{{{\partial }^{2}}}}{{\partial {{x}^{2}}}}{{f}_{\eta }}(x)} \right]}}^{2}}dx} $
для классических “непрерывных” ядерных статистических оценок плотности (см., например, [9], [10]). В связи с этим возникла идея попробовать использовать следующую двухстадийную технологию выбора параметров $M$ и $n$ в одномерном случае.

Сначала выбираем параметр $M = \tilde {M}$ по формулам

(7.13)
$2 \times 16 \times \tilde {H}_{{{\text{bias}}}}^{{({{\mathbb{L}}_{2}})}}{{M}^{{ - 4}}} = \frac{{{{L}^{2}}}}{2}\quad {\text{или}}\quad \tilde {M} = \frac{{{{H}^{{(h \to M)}}}}}{{\sqrt {3L} }}{{\left( {\int\limits_X {{{{\left[ {\frac{{{{\partial }^{2}}}}{{\partial {{x}^{2}}}}{{f}_{\eta }}(x)} \right]}}^{2}}} dx} \right)}^{2}}.$
Здесь учтено соотношение (2.8), а также то, что $\delta _{{{\text{det}}}}^{{({{\mathbb{L}}_{2}})}}(M)$ примерно в три раза больше $\delta _{{{\text{bias}}}}^{{({{\mathbb{L}}_{2}})}}(M)$ (см. соотношения (3.9), (4.9)).

Далее рассматриваем уравнение

(7.14)
$\tilde {\delta }_{{{\text{stoch}}}}^{{({{\mathbb{L}}_{2}})}}(\tilde {M},\tilde {n}) = \frac{{{{f}_{{{\text{max}}}}} \times \operatorname{mes} X \times \tilde {M}}}{{{{H}^{{(h \to M)}}}\tilde {n}}}\quad {\text{или}}\quad \tilde {n} = \frac{{2{{f}_{{{\text{max}}}}} \times \operatorname{mes} X \times \tilde {M}}}{{{{H}^{{(h \to M)}}}{{L}^{2}}}}.$

Мы сравнили формулы (7.3), (7.4) (для $d = 1$) и (7.13), (7.14) на тестовом примере приближения плотности ${{f}_{\eta }}(x) = \tfrac{{{{e}^{{1 - x}}}}}{{e - 1}}$, $0 < x < 1$. При этом выборочные значения случайной величины $\eta $ моделировались по формулам ${{\eta }_{j}} = - ln[1 - {{\alpha }_{j}}(1 - {{e}^{{ - 1}}})]$, где ${{\alpha }_{j}} \in U(0,1)$ – стандартные случайные числа (см., например, [19]).

Для уровня погрешности $L = 0.003$ формулы (7.3) и (7.4) дали значения $M_{{{\text{opt}}}}^{{({{\mathbb{L}}_{2}})}}(L) = 14$, $n_{{{\text{opt}}}}^{{({{\mathbb{L}}_{2}})}}(L) = 3\,256\,877$ (максимальная погрешность приближения функции ${{f}_{\eta }}(x)$ для таких параметров получилась равной $0.002871 \lesssim L$, а время счета ${{t}_{{{\text{opt}}}}} = 0.446$ с). Аккуратные массовые расчеты показали оптимальность полученных значений (т.е. при других сочетаниях параметров $M$ и $n$ были превышены либо уровень погрешности $L$, либо время счета ${{t}_{{{\text{opt}}}}}$).

Формулы (7.13) и (7.14) для рассмотренного примера и для $L = 0.003$ дали значения $\tilde {M} = 11$ и $\tilde {n} = 3\,779\,382$. Максимальная погрешность приближения функции ${{f}_{\eta }}(x)$ для таких параметров получилась равной $0.001696$ (что заметно меньше заданного уровня $L$), а временные затраты $0.542$ с (что заметно больше величины ${{t}_{{{\text{opt}}}}}$).

8. ЗАКЛЮЧЕНИЕ

В данной работе предложен новый конструктивный ядерный алгоритм численного функционального приближения вероятностной плотности ${{f}_{\eta }}({\mathbf{x}})$ (алгоритм 1) по заданной выборке, основанный на использовании ядерной оценки (1.4) и на подходах теории численной аппроксимации функций. Использовано соображение из [1]–[4] о том, что построенный ядерный алгоритм 1 аналогичен функциональному вычислительному ядерному статистическому алгоритму для приближения решения интегрального уравнения Фредгольма II рода (см. замечание 1). В связи с этим в [1]–[4] было предложено использовать схему условной оптимизации функциональных алгоритмов, основанную на минимизации затрат $S(M,n)$ при фиксированном уровне погрешности $L$ (метод 1), для функционального ядерного алгоритма 1. В данной работе получен вид условно-оптимальных параметров (7.3), (7.4) и (7.10), (7.12) (для известных ${{\mathbb{L}}_{2}}$- и $\mathbb{C}$-подходов соответственно) для простейшего случая алгоритма 1 с аппроксимационным базисом вида (3.1) с образующей функцией (3.4), простейшими аппроксимационными коэффициентами вида (3.6) и кусочно-постоянной ядерной функцией вида (4.5) (т.е. для многомерного аналога алгоритма метода полигона частот).

Список литературы

  1. Булгакова Т.Е., Войтишек А.В. Сравнительный анализ функционального “ядерного” алгоритма и метода полигона частот // Тр. Международ. конф. “Актуальные проблемы вычислительной и прикладной математики” (ИВМиМГ СО РАН, Новосибирск, 1–5 июля 2019 г.). Новосибирск: ИВМиМГ СО РАН, 2019. С. 65–71.

  2. Булгакова Т.Е., Войтишек А.В. Критерии оптимизации “ядерного” алгоритма приближения вероятностной плотности // Тр. XV Международ. Азиатской школы-семинара “Проблемы оптимизации сложных систем” (26–30 августа 2019 г., РФ, Новосибирск). Новосибирск: ИВМиМГ СО РАН, 2019. С. 15–23.

  3. Voytishek A.V., Bulgakova T.E. On conditional optimization of “kernel” estimators of densities // Proceed. Fifth Inter. Workshop “Applied Methods of Statistical Analysis. Statistical Computation and Simulation” (Novosibirsk, Russia, 18–20 September, 2019). Novosibirsk: NSTU Publ., 2019. P. 152–159.

  4. Voytishek A.V., Bulgakova T.E. Optimization of kernel estimators of probability densities // Jacimovic M., Khachay M., Malkova V., Posypkin M. (eds). Optimizat. and Appl. OPTIMA 2019. Comm. Comput. and Informat. Sci. 2020: Springer, Cham. V. 1145. P. 254–266.

  5. Лемешко Б.Ю., Лемешко С.Б., Блинов П.Ю. Критерии проверки статистических гипотез при анализе больших выборок: проблемы и их решение // Тр. Международ. конф. “Актуальные проблемы вычислительной и прикладной математики” (ИВМиМГ СО РАН, Новосибирск, 1–5 июля 2019 г.). Новосибирск: ИВМиМГ СО РАН, 2019. С. 277–283.

  6. Lemeshko B.Yu., Lemeshko S.B., Semenova M.A. Features of testing statistical hypotheses under big data analysis // Proceed. Fifth Inter. Workshop “Applied Methods of Statistical Analysis. Statistical Computation and Simulation” (Novosibirsk, Russia, 18–20 September, 2019). Novosibirsk: NSTU Publ., 2019. P. 122–137.

  7. Епанечников В.А. Непараметрическая оценка многомерной плотности вероятности // Теория вероятностей и ее применения. 1969. Т. 14. Вып. 1. С. 156–161.

  8. Бахвалов Н.С. Численные методы. М.: Наука, 1975.

  9. Михайлов Г.А. Рандомизированные алгоритмы метода Монте-Карло для задач со случайными параметрами (метод “двойной рандомизации”) // Сиб. журн. вычисл. матем. 2019. Т. 22. № 2. С. 187–200.

  10. Михайлов Г.А. Улучшение многомерных рандомизированных алгоритмов метода Монте-Карло с “расщеплением” // Ж. вычисл. матем. и матем. физ. 2019. Т. 59. № 5. С. 822–828.

  11. Войтишек А.В. Лекции по численным методам Монте-Карло. Новосибирск: ИПЦ НГУ, 2018.

  12. Mikhailov G.A. Minimization of Computational Costs of Non-Analogue Monte Carlo Methods // Ser. Sov. East Europ. Math. V. 5. Singapore: World Sci., 1991.

  13. Mikhailov G.A. New Monte Carlo Methods with Estimating Derivatives. Utrecht: VSP, 1995.

  14. Войтишек А.В. Основы метода Монте-Карло в алгоритмах и задачах. Ч. V. Вычисление многократных интегралов. Аппроксимация интегралов, зависящих от параметра. Новосибирск: НГУ, 1999.

  15. Михайлов Г.А. Весовые методы Монте-Карло. Новосибирск: Изд-во СО РАН, 2000.

  16. Шкарупа Е.В. Сходимость и оптимизация численных дискретно-стохастических процедур: Дис. … канд. физ.-мат. наук / НГУ. Новосибирск, 2000.

  17. Войтишек А.В. Дискретно-стохастические численные методы: Дис. … д-ра физ.-мат. наук / РАН. Сиб. отд-ние, ИВМиМГ. Новосибирск, 2001.

  18. Войтишек А.В. Основы метода Монте-Карло в алгоритмах и задачах. Ч. VI. Вычисление значений линейных функционалов от решения интегрального уравнения второго рода. Дискретно-стохастические методы решения интегрального уравнения второго рода. Новосибирск: НГУ, 2004.

  19. Михайлов Г.А., Войтишек А.В. Численное статистическое моделирование. Методы Монте-Карло. М.: Изд. центр “Академия”, 2006.

  20. Милосердов В.В. Дискретно-стохастические численные алгоритмы со сплайн-восполнениями: Дис. … канд. физ.-мат. наук. Новосибирск, 2006.

  21. Войтишек А.В. Функциональные оценки метода Монте-Карло. Новосибирск: НГУ, 2007.

  22. Шкарупа Е.В. Оценка погрешности и оптимизация метода полигона частот для глобального решения интегрального уравнения II рода // Ж. вычисл. матем. и матем. физ. 1998. Т. 38. № 4. С. 612–627.

  23. Shkarupa E.V., Voytishek A.V. Convergence of discrete-stochastic numerical procedures with independent or weakly dependent estimators at grid nodes // J. Statist. Plann. and Infer. 2000. V. 85. P. 199–211.

  24. Войтишек А.В., Головко Н.Г., Шкарупа Е.В. Оценка погрешности многомерного аналога метода полигона частот // Сиб. ж. вычисл. матем. 2002. Т. 5. № 1. С. 11–24.

  25. Марчук Г.И., Агошков В.И. Введение в проекционно-сеточные методы. М.: Наука, 1981.

  26. Бахвалов Н.С., Лапин А.В., Чижонков Е.В. Численные методы в задачах и упражнениях. М.: Высш. школа, 2000.

  27. Литбеттер М., Ротсен Х., Лингрен Г. Экстремумы случайных последовательностей и процессов. М.: Мир, 1989.

Дополнительные материалы отсутствуют.